Vous êtes sur la page 1sur 48

QUANT REPLICA QUESTIONS THAT HAVE APPEARED IN

CAT IN THE LAST 4 YEARS


Permutations and Combinations

5.

For any N, the number of non-mates of each


member of X is
1 2
(1)
(N 3N + 2)
(2) 2N 5
2
1 2
(N 5N + 6)
(3) 3N 10
(4)
2
1 2
(5)
(N 7N + 16)
2

6.

For any N, the number of common mates of


two mates in X is
(1) N 2
(2) 3N 8
1 2
(N 6N + 12)
(4) N 1
(3)
2
1 2
(5)
(N 3N)
2

Directions for question 1: Select the correct


alternative from the given choices.
1.

X = {0, 1, 2, 3, 5}
I is the set of integers greater than 999 and not
exceeding 5000, formed by using one or more
elements of X as its digits. Find the number of
elements in I.
(1) 376
(2) 375
(3) 500
(4) 499
(5) 501

Directions for questions 2 and 3: Answer these


questions based on the following information.
The figure below is the plan of a town T. The streets
are at right angles to each other. Shoba is a resident
of T whose house is at H and whose office is at O.
R is a rectangular park in which there is a road along
the diagonal AB and P is a prohibited region in T.
C

H
A

Directions for questions 7 to 19: Select the correct


alternative from the given choices.
7.

How many five-digit numbers divisible by 9 can


be formed using the digits 0, 1, 2, 4, 6, 8, 9 such
that each digit occurs atmost once in any such
number?
(A) 96
(B) 192
(C) 288
(D) 144

8.

How many five-digit numbers divisible by 4 can


be formed using the digits 0, 7, 5, 2, 4, 6 without
repeating any digit?
(A) 126
(B) 186
(C) 204
(D) 180

9.

Six foot ball players, seven hockey players, eight


tennis players and nine cricket players are to be
arranged in a row. In how many can they be
arranged such that all the players who play the
same game sit together?
(A) 6!7!8!9!
(B) 4!
(C) 30!
(D) 4!6!7!8!9!

P
R
B

O
2.

Shoba wants to go from H to O taking the shortest


path. How many paths can she choose from?
(1) 60
(2) 108
(3) 90
(4) 126
(5) 72

3.

Shoba wants to go from H to the club C, shown in


the figure through O, taking the shortest path.
How many paths can she choose from?
(1) 1638
(2) 936
(3) 1170
(4) 1404
(5) 1560

Directions for question 4: Select the correct


alternative from the given choices.
4.

K = (p + q + r)15. Find the number of distinct terms


in the expansion of the bracket.
(1) 136
(2) 153
(3) 145
(4) 120
(5) 128

Directions for questions 5 and 6: Answer these


questions based on the information below.
X is the set of all pairs (q, p) where p and q satisfy
N p > q 1, where N 4. If any two distinct members
in X have one number in common, they are called
mates. Otherwise they are called non-mates. For
example if N = 4, then X = {(3, 4), (2, 3), (2, 4), (1, 2),
(1, 3), (1, 4)}. In X, (1, 3) and (1, 4) are mates (2, 3)
and (3, 4) are also mates but (3, 4) and (1, 2) are
non-mates.

10. In how many ways can seven students be sent


into five different sections?
(A) 75
(B) 57
(C) 35
(D) None of these
11. The sum of all the five-digit numbers that can be
formed by using the digits 2, 4, 6, 7, 8 without
repetition is
(A) 7919982
(B) 7199928
(C) 7999182
(D) 7919928
12. There are 40 lines in a plane of which a set of
12 lines are concurrent at A, another set of
15 lines are concurrent at B and the set of
remaining lines are parallel. What is the number
of points of intersection of these 40 lines, given
that the three sets are disjoint?
(A) 611
(B) 531
(C) 533
(D) 638
13. The coefficients of how many terms in the
expansion of (x + y z)100 are negative?
(A) 2450 (B) 2500 (C) 2550 (D) 2600

2.

ABC is an obtuse angled triangle whose sides


are 7 cm, 16 cm and y cm where y is an integer.
Find the number of possible values of y.
(1) 9 (2) 10 (3) 6 (4) 16 (5) 17

3.

15. How many 5 - digit numbers, comprising only the


digits 4 and 5 are divisible by 3?
(A) 10
(B) 11
(C) 12
(D) More than 12

PQRS is a square. M and N are the midpoints of


PS and QR respectively. X and Y are points lying
on the line joining M and N inside PQRS, such
that PXS = QYR = 60. Find the ratio of the
area of the hexagon PQXRSY and the remaining
area inside PQRS.

16.

(1) 4

(2) 2 3 + 3

(4) 1 +

(5)

14. All possible words are formed using all the letters
in the word EQUATION exactly once and are
arranged such that no two vowels and no two
consonants are in alphabetical order. How may
words are there?
(A) 56
(B) 140
(C) 720
(D) 4914

4.

There is a rectangular grid consisting of unit cells


as shown in the figure given below. One needs to
travel from point A to point C. One can travel only
to the right and upwards. In how many ways can
one go from point A to point C without traveling
via B?
(A) 3915
(B) 3150
(C) 2520
(D) 6434

19. Three, four and five points are selected on the


sides AB, BC and CD respectively of ABC.
None of the 12 points coincide with A, B and C.
How many triangles can be formed by using
these 12 points as vertices?
(A) 220
(B) 217
(C) 210
(D) 205

1.

PQR is a triangle. PQ = 19.5 cm and QR = 11 cm.


PS is the altitude of PQR of length 5 cm. C is a
circle circumscribing the triangle PQR. Find the
radius (in cm) of C.
(1) 18.05
(2) 28.85
(3) 27.25
(4) 21.45
(5) 31.25

2
3

3
2

5.

The radius of the base of a right circular cone is


6 cm and the height is 21 cm. The cylinder having
the maximum possible total surface area is
placed inside the cone such that one of its flat
surfaces rests on the base of the cone. Find the
total surface area of the cylinder, in sq.cm.
1321
1353
1383
(1)
(2)
(3)
8
8
8
1473
441
(5)
(4)
8
5

6.

C1 and C2 are two equal circles whose centers


are M and N respectively. They intersect at X and
Y. Neither of the centers lies inside the other
circle. If XMN = , what is the range of possible
value of ?
(1) 0 30
(2) 0 45
(3) 0 60
(4) 0 75
(5) 0 90

7.

Two identical circles intersect each other.


The radii of the circles and the distance joining
the centres of both the circles are in the ratio of
1 : 3 . Find the ratio of the area of the region
common to both the circles, to that of a circle.

Geometry & Mensuration


Directions for questions 1 to 40: Select the correct
alternative from the given choices.

3 +1

Each of two circles passes through the centre of


the other. The radius of either circle is 2 cm.
Find the area of the intersecting region (in sq.cm).
4
8
(1)
3
(2)
+2 3
3
3
8
8
2 3
(4)
+ 3
(3)
3
3
(5)

17. How many three-digit odd number can be formed


such that if 2 is one of the digits, the following
digit is 4?
(A) 5
(B) 365
(C) 300
(D) 372
18. A coaching institute was to send 9 parcels for its
postal students, four of whom were from Bihar
and the remaining five were from Delhi. The
dispatch clerk made a mistake in addressing the
parcels. He addressed them in such a way that
no student received the correct parcel although
all the parcels for Bihar were sent to Bihar and all
the parcels for Delhi were sent to Delhi. Find the
total number of ways in which he could have
addressed those 9 parcels.
(A) 396
(B) 2737 (C) 475
(D) 89496

(3) 2 3 1

8.

(A)

3
4

(B)

3 2 3
12

(C)

3
6

(D)

2 3 3
6

A large number (greater than 1200) of equilateral


triangles of side 2 units are available to form
equilateral triangles. Equilateral triangles of sides
16 units, 18 units, 30 units are formed using
the available triangles. Find the total number of
triangles used.
(A) 1050 (B) 1100 (C) 1150 (D) 1200

9.

Find the side of the smallest equilateral triangle


that can contain a semicircle of radius 8 3 cm.
(in cm)
(A) 16 3

(B) 32

(C) 32 3

(D) 40

10. In the figure below, ABC is


equilateral. Three identical
circles are shown in it. Each
circle touches the other two
circles and has two of the
sides of the triangle being
tangents to it. Find the ratio of the side of the
triangle and the radius of each circle.

( )
2(1 + 3 )

(
)
2(1 + 3 3 )

(A) 2 2 + 3

(B) 2 1 + 2 3

(C)

(D)

11. N points are marked on the circumference of a


circle. The number of triangles which can be
drawn by using these points as vertices is 210
more than the number of triangles which can be
drawn by using all but one of these points as
vertices. Find N.
(A) 21
(B) 22
(C) 23
(D) 24
12. Let X denote the product of the sides of a triangle.
Let Y denote the product of its semi-perimeter,
Y
=
circumradius and its inradius.
X
1
1
(A)
(B)
4
6
1
(D) Cannot be determined
(C)
3
13. In the figure above, PQRS P
is a square. A sector of a
circle with centre S is
shown.
TQVU
is
a
rectangle such that TU = 9
and UV = 32. Find PS.
(A) 60
S
(B) 65
(C) 70
(D) Cannot be determined

T
U

Q
V

14. Two intersecting circles are said to be orthogonal,


if a pair of tangents drawn to them at any point of
intersection are perpendicular to each other.
C1 and C2 are orthogonal circles. Each of them
has a radius of 8 cm. Find the area of the region
common to both the circles. (in sq cm)
(A) 16(2 1)
(B) 16(2 3)
(C) 32( 1)
(D) 32( 2)
15. On the periphery of a square grass field ABCD,
two poles are fixed. One of them is fixed at the
midpoint of side AB while the other one is fixed at
the midpoint of side BC. Side of ABCD is 16 m.
A cow is tied to one of the poles and another is
tied to the other pole. If each cow is tied with a
8 m long rope, then find the ungrazable area in
the field. (in sq. m)
(A) 256 32
(B) 216 32
(C) 192 32
(D) 240 32

16. PQR is an isosceles triangle with PQ = QR. QA is


the median to PR. B is a point on QR (or QR
extended) such that AQ = AB. PQA = 30.
Find ABQ.
(A) 20
(B) 30
(C) 40
(D) Cannot be determined
17. A triangle has an area of 960 sq m. Two of its
sides are 68 m and 32 m. Find its third side if it is
given that the triangle is right angled (in m).
(A) 52
(B) 58
(C) 62
(D) None of these
18. A right circular cone has a radius of 18 cm and a
height of 54 cm. From it, a right circular cylinder
of radius 15 cm was cut. Find the volume of the
conical part of the remaining solid (in cubic cm).
(A) 4540 (B) 4375 (C) 4780 (D) 3375
19. Two plastic cubes each of which have an integral
side (in cm), have the sum of their lateral surface
areas equal to 468 sq cm. The sum of their
volumes is 945 cubic cm. Find the volume of the
hemisphere whose radius equals the sum of the
edges of the two cubes (in cubic cm)
(A) 2150 (B) 2250 (C) 2325 (D) 2400
20. In ABC, AB = 32, BC = 24 and AC = 40. Using
B as a centre, a circle is drawn. This circle cuts
AB and BC at D and E respectively. The radius of
this circle equals the circumradius of ABC.
Find AD : EC.
(A) 3 : 1
(B) 2 : 1
(C) 3 : 2
(D) 5 : 2
21. There are 13 spherical balls, each of radius
3 2 . Nine of the balls are arranged in layer in a
3 x 3 square formation. Four balls are placed in a
second layer over the four depressions which are
formed. Find the height of the smallest cuboid
which can enclose these balls.
(A) 6 + 6 2

(B) 6+6 3

(C) 2 3 +2 6

(D) 3 2 +6

22. Four points are chosen at random in a region


comprising an equilateral triangle of side
4 3 and its interior. Which of the following is
true?
(A) There are at least two points P, Q such that
PQ 4.
(B) There are at least two points P, Q, such that
PQ 4.
(C) There are at least two points PQ, such that
PQ > 4.
(D) There are at least two points PQ, such that
PQ < 4.
23. In the figure below, PQRS is a rectangle with
PQ = 9 and PS = 6. MNOP is a smaller rectangle
with MN parallel to PQ. Also the lengths of MN
and MP are integers and A, B, C, D are the
midpoints of the sides of the rectangle MNOP
shown in the figure. The area of the shaded
2
region is 16 /3% of the area of the rectangle
PQRS. Find the length of AB.

P
M

N
B

D
P

S
(A)

29. In the figure below, circles with centres A, B and


C have equal radii and similarly circles with
centres D, E and F have equal radii. Find the ratio
of the radii of circles with centres at D and G.

Q
A

(B) (3 / 2) 5

25. In the triangular field PQR, PQ = 54 m, QR = 240 m


and Q =900. [PQ, AB, CD, EF, GH, and IJ are
all equispaced and all the lines are perpendicular
to QR. Also HI = IR]. If one moves along
PQABCDEFGHIJR, then what is the total
distance covered (in meters)?
P

B
C
F
G

Q A D E
(A) 270m
(C) 351m

H I

(B) 594m
(D) None of these

26. The maximum number of acute angles in a


convex octagon is
(A) 1
(B) 2
(C) 3
(D) None of these
27. There are 6 boys B1, B2, B3, .B6 standing
at the vertices A to F respectively of a regular
hexagon. They start walking simultaneously
along the perimeter in the clockwise direction with
speeds in the ratio
1 : 2 : 3 : 4 : 5 : 6. When
B6 completes 3 full rounds, which are the vertices
at which there are no boys?
(A) A, C
(B) B, E
(C) A, C, E
(D) B, D
28. On square ABCD, points P, Q, R and S are on
sides DA, AB, BC, CD such that AP = AQ = CR =
CS X and Y are points on AB and CD
respectively, which are equidistant from P and R.
XB + YD
Find
AP + CR
(A) 0.5
(B) 0.75
(C) 1
(D) Cannot be determined

(A)

13 + 10 3
11

(B)

17 + 10 3
11

(C)

19 10 3
11

(D)

10( 2 + 2)
11

30. Which of the following statements is/are true?


() A triangle exists with altitudes measuring 6,
9 and 15
() A triangle exists with altitudes measuring 4,
9 and 12 which of the following is true.
(A) Only
(B) Only II
(C) Both and
(D) Neither I or
31. A, B, C and D are four friends who are standing
at the four corners of a rectangular field in the
anticlockwise order. They decide to meet at a
point P inside the field. If the shortest distance of
the point P from the initial position of A, B, C, and
D are 20 m, 40 m, 60 m and x m respectively,
then find the value of x.
(A) 30 2

(C) 2 5
(D) Cannot be uniquely determined
24. There are two concentric circles with their centre
at O. Chord AB of the outer circle is tangent to
the inner circle. If the measure of the line AB is
40 cm and the radii of both the inner and the
outer circle are integral number of centimeters,
then which of the following cannot be the length
of the diameter of the outer circle?
(A) 50
(B) 58
(C) 96
(D) 104

(B) 20 6

(C) 40 3

(D) 50

32. When a rectangle ABCD (with AB < BC) is folded


such that the vertex C touches vertex A, the
length of the crease XY formed is equal to the
length of the rectangle. How many times the
length of the rectangle is its breadth?
2 + 1

(A)
2

5 + 1

(C)
2

1
2

2 1

(B)
2

1
2

5 1

(D)
2

1
2

1
2

33. Two perpendicular chords PQ and RS intersect at


T PT= 4 and QT=18. If TS is 2 times TR, then
find the radius of the circle
(A)

110

(B) 2 30

(C)

130

(D) 5 6

34. On side AB of rectangle ABCD, P is a point such


that APD : DPC: CPB = 2 : 5 : 5. Find AB :
BC.
(A) 3
(C) 2

(B) 2 3 2
(D) Cannot be determined

35. A cuboid has a square base. The length of its


longest diagonal is

2502 . All dimensions of the

cuboid are integral. How many such cuboids are


possible?
(A) 1
(B) 2
(C) 3
(D) 4
36. In a cyclic quadrilateral, the diagonals AC and BD
intersect at P at right angles. If PA = 8, PC = 9
and the radius of the circumcircle of ABCD is
81.25 , then find PD given that where PD < PB.
(A) 12
(B) 9
(C) 8
(D) 6

37. In trapezium ABCD, the diagonals AC and BD


intersect at P. Three times the area of the
pentagon APBCD is equal to 7 times the area of
PCD. Find AP : PC.
3
4
2
3
(A)
(B)
(C)
(D)
3
4
3
2
38. The semiperimeter of a right angled triangle is
60 and the inradius is 8. Find the length of the
smaller leg.
(A) 30
(B) 20
(C) 24
(D) 18
39. There are two vertical poles AB and CD where
B and D are on the ground. AB = 4, CD = 7.5,
BD = 11.5. On line BD, P is a point where the
points A and C subtend a right angle and Q is a
point that is equidistant from A and C. Find PQ.
(A) 3
(B) 3.5
(C) 4
(D) Cannot be determined

3. Nagu bought a new bike and went for a drive,


liding at a uniform speed on a highway. At 9 am
he passed a milestone. He turned his head back
and read the number on the milestone.
He continued driving and at 10 am, he passed
another milestone. Again he turned his head back
and saw that the number was the reverse of the
number on the first milestone. He continued
driving and at 11 am he passed a third milestone.
Again, he turned his head back to note the
number. The sum of the digits of the number on
this milestone was equal to the sum of the digits
of the number on the first milestone. Which of the
following is the speed at which he travelled?
(Assume that the numbers he saw were
increasing).
(A) 9 km/hr
(B) 45 km/hr
(C) 18 km/hr
(D) Cannot be determined.

Equations, Ratio, Proportion, Variation


Directions for questions 1 to 6: Select the correct
alternative from the given choices.
1.

Amar, Bhuvan, Chetan and Dinesh are


four friends. Amar has m marbles with him.
He gives Bhuvan 1 less than half the number of
marbles he has. Then he gives Chetan 1 less
than half the remaining number of marbles he
has. Finally, Amar gives Dinesh 1 less than half
the remaining number of marbles he has and is
left with 4 marbles. Which of the following best
describes the value of m?
(1) 1 m 4
(2) 5 m 9
(3) 9 m 13
(4) 10 m 14
(5) m 14

2.

If a

3.

If 3a + 5b + 7c = 1.25 k and 2a + b + 3c = 0.75 k,


then 7b + 5c is what percentage of k?
(A) 25%
(B) 50%
(C) 35%
(D) 75%

4.

There are some two rupee coins and five rupee


coins in a bag. If the number of five rupee coins is
tripled, then the amount in the bag is increased
by 75%. Which of the following can be the
number of five rupees coins in the bag?
(A) 13
(B) 20
(C) 18
(D) 32

5.

There are 100 questions in a test paper. Four marks


are awarded for each right answer and two marks
are deducted for each wrong answer. If Abhilash
attempts more than 85 questions and get
70 marks, What is the minimum number of
questions that he could have answered correctly?
(A) 40
(B) 38
(C) 39
(D) 41

0
40. In triangle PQR, Q =90 and PQ = QR. D is a
point on the same side of PR as Q such that PR

=
(A)
(B)
(C)
(D)

2 QD. Find the measure of angle PDR.


1
22 /2
30
45
Cannot be uniquely determined

Special Equations
Directions for questions 1 to 3: Select the correct
alternative from the given choices.
1.

2.

In a certain country, the currency used was called


femto. Raju had his lunch in one of the hotels in it.
His bill came to 47 femtos. He had notes in only
3 denominations 1-femto, 2-femto and 20-femto.
In how many ways can he settle the bill?
(1) 24 (2) 18 (3) 48 (4) 16 (5) 42
A bank teller was making the payment for a
cheque presented by Mohan. As he was in a
confused state of mind, he transposed the rupees
and paise and hence gave more than what he
should have. Mohan left the bank and bought a
biscuit from a nearby store for `1.50.
The amount remaining with him was 4 times the
amount on the cheque. The amount remaining
with him must have been between
(1) `70 and `71
(2) `79 and `80
(3) `85 and `94
(4) ``93 and `94
(5) `98 and `99

1
and b c2, then find the value of
b
9a + 5c, when b = 27. Given that when b = 3 then
a = 4 and c = 6.
(A) 94
(B) 49
(C) 76
(D) 38

6.

Arun, Varun and Kiran have a total of `8,000 with


them. They spend `50, `100 and `200
respectively, Now they have a money is the ratio
of 14 : 22 : 15. What is the amount with the Varun
initially?
(A) 2150
(B) 3400
(C) 3300
(D) 2100

5.

In an x-metre race, A beats B by 180 m and


C beats by 351 m. In the same race, B beats
C by 198 m. Find x?
(A) 1200 (B) 1000 (C) 1122 (D) 1320

6.

Towns A and D are 36 km apart. Three friends,


Tarun, Varun and Arun start together from
A towards D. While Arun sets off on foot, Varun
takes Tarun along on his bike and travels at
45 km/hr. He drops Tarun at a point C and turns
back for Arun. He meets Arun at a point B, and
then turns back towards D. All the three friends
reached D together. If both Arun and Tarun
walked at 5 km/hr, how long do the 3 take to
cover AD?
2
1
13
(A) 3
(B) 3
(C) 3
(D) 1
3
9
15

Time and Distance


Directions for question 1: Select the correct
alternative from the given choices.
1.

P and Q are 2 stations. Raju plans to drive from


city R, located 840 km directly to the north of Q,
210
kmph so that he can
to Q, at a speed of
3
catch a train arriving there from P. The train
leaves P at 9 a.m. and travels at a speed of
84 3 kmph. P is between east and south-east
of R with RQ at 60 to RP. Also P is between the
north and the north-east of Q with PQ at 30 to
RQ. If Raju has to reach P at least 25 minutes
before the train, then find the approximate latest
time at which he should start from R.
(1) 6:50 a.m. (2) 6:40 a.m. (3) 6:20 a.m.
(4) 6:30 a.m. (5) 7:00 a.m.

Directions for questions 2 and 3: Answer these


questions based on the information below.
Cities P and Q are 4800 km apart. P is to the west of
Q. Both cities are in different time zones Best
Airlines was an airline which operates non-stop
flights between the cities. All its planes cruise at the
same speed in both directions. However, the effective
speed of any plane is influenced by a steady wind
blowing from west to east at 100 kmph. The table
below shows the departure time of the planes from
each city and their arrival time at the other city.
(Given below are local times of the respective cities)
Departure
City
Time
P
7:00 a.m.
Q
6:00 p.m.

City
Q
P

Arrival
Time
5:00 p.m.
4.00 a.m.

2.

Find the time in P when the plane landed in Q.


(1) 4:00 p.m.
(2) 3:00 p.m.
(3) 3:30 p.m.
(4) 2:30 p.m.
(5) Cannot be determined

3.

Find the cruising speed of the plane (in kmph).


(1) 600
(2) 550
(3) 500
(4) 450
(5) Cannot be determined

Directions for questions 4 to 8: Select the correct


alternative from the given choices.
4.

A starts from point P at 9:00 a.m. and travels east


at 45 km/hr. B starts a bit later from P and travels
south at 30 km/hr for 48 minutes. At that instant,
he is 74 km from A. When does B start from P?
(A) 9:42 a.m.
(B) 9:45:20 a.m.
(C) 9:48 a.m.
(D) 9:51:40 a.m.

7. Two friends P and Q start simultaneously from


the opposite ends of a race track AB of length
150 m at speeds 60 m/s and 40 m/s respectively.
P starts from A and Q starts from B. Once each
reaches an end, he immediately turns back and
moves towards the other end. They keep moving
to and from between the two ends. Find the
difference between the distance covered by P
and Q by the 8th meeting.
(A) 300 m (B) 450 m (C) 600 m (D) 1200 m
8. Ajay and Sanjay start simultaneously from the
same point on a circular track. If they travel in
opposite directions, they meet at 7 distinct points
on the track whereas if they travel in the same
direction, then they meet at n distinct points on it
where n is a prime number. If Ajay is faster than
Sanjay and Sanjay's speed is P% less than that
of Ajay, then which of the following can be a
value of P?
1
(A) 25
(B) 50
(C) 75
(D) 83 /3

Quadratic Equations
Directions for questions 1 and 2: Select the correct
alternative from the given choices.
1.

Three consecutive positive integers are taken in


descending order. The first, second and the third
are raised to the first, second and third powers
respectively. The powers are added and the
result is multiplied by 3. The square root of the
result is found to be the sum of the original
integers. The least of the integers is denoted by
L. Which of the following holds true?
(1) 1 L 4
(2) 5 L 8
(3) 9 L 12
(4) 13 L 15
(5) L > 15

2.

The roots of the equation x3 px2 + qx r = 0 are


a, b and c, which are consecutive integers.
Find the least possible value of q.
(1) 0

(2) 1

(4) 1

(5) 2

(3)

Directions for questions 3 and 4: Answer these


questions using the information below.
2

Let g(x) = px + qx + r, where p, q and r are constants


and r 0. One root of g(x) = 0 is 4 and 8g(3) = 3g(6).
3.

4.

Find the other root of g(x) = 0.


(1) 1
(2) 3
(3) 6
(4) 2
(5) Cannot be determined
Find the sum of p, q and r.
(1) 21
(2) 20
(3) 19
(4) 18
(5) Cannot be determined

Directions for questions 5 and 6: Answer these


questions based on the given information.

12. A certain number of cups of tea are available for


`90. If the price of each cup increases by `1.50,
the number that can be bought for the same
amount decreases by 10. Find the actual cost of
each cup of tea (in rupees).
(A) 3
(B) 3.50
(C) 4
(D) 4.50
13. The expression x4 + y4 2x2 y2 32x2 32y2 +
256 is completely factorized into real factors.
Which of the following statements about the
factors is true?
(A) There are 2 irreducible quadratic factors.
(B) There is a cubic factor and a linear factor.
(C) There are 4 linear factors.
(D) None of these
2

Raju makes and sells an item in a market every day.


He sells each unit of that item at `106. The cost of
2
producing x units per day is 200 + px + qx , where p
and q are constant. If Raju increased his daily
production from 10 units to 15 units, his daily
8
production cost would increase by 88 /9%. If he
increased his daily production from 15 units to
20 units, his daily production cost would increase by
12
64 /17%. Assume that there is a high demand for the
item and that Raju can sell whatever he produces.
He wishes to maximize his profit.

14. The expression ax + bx + c takes a maximum


value of 5 at x = 1 and takes the value of -1 at
x = 0. Find the value of the expression at x = 5.
(A) 89
(B) 91
(C) 210
(D) 211

5.

Rajus daily production must be (in units).


(1) 10 (2) 15 (3) 8 (4) 5 (5) 12

6.

Find the maximum daily profit that Raju can


obtain (in Rs).
(1) 400
(2) 425
(3) 350
(4) 375
(5) 184

16. If the sum of the roots (not necessarily real) of a


quadratic equation is 6 and the sum of the
squares of the roots is 16, then find the product of
the roots.
(A) 8
(B) 10
(C) 9
(D) Such an equation doesn't exist

Directions for questions 7 to 18: Select the correct


alternative from the given choices.
7.

E(y) is a quadratic expression. It has the


minimum value of 1 when y = 3 and E(2) = 2.
Find E(4).
(1) 40 (2) 45 (3) 50 (4) 55 (5) 60

8.

If the equations ax2 + bx + c = 0; bx2+ cx + a = 0


have one root in common, then which of the
following is definitely true?
3
3
(A) a + b = 3abc
(B) a3 + b3 + c3 = 3abc
3
3
3
(C) a b c = 3abc
2
2
(D) a + b + c2 = 2ab + 2bc + 2ca

9.

If , and are the roots of the equation


3
2
x + 2x 5x 6 = 0 and , , are the roots of
3
2
the equation x + px + qx + r = 0; then the value of
p is
(A) 6
(B) 5
(C) 5
(D) 6

10. Let and be the roots of a quadratic equation


and = 9 and || || = 5 then find the
product of roots.
(A) 14
(B) 21
(C) 16
(D) 7
11. What is the minimum value of the square of the
difference of the roots of the quadratic equation
x2 (k + 7) x (3k 15) = 0?
(A) 160
(B) 180 (C) 160 (D) 180

15. The roots of the equation x3 + ax2 + bx + c = 0,


where c > 0, are k, kr, kr2 where k and r are
integers. If the sum of the squares of the roots is
364, then find the value of c
(A) 180
(B) 72
(C) 108
(D) 216

17. The roots of x3 21x2 + px 280 = 0 are in


arithmetic progression. Find the value of p.
(A) 138
(B) 128
(C) 118
(D) Cannot be determined
18. If cubic equation ax3 + bx2 + cx + d = 0 has two
positive roots where a, b, c, d are real and d0,
then which of the following is true?
(A) c and b are of opposite signs.
(B) a and d are of the same signs.
(C) b and c are of the same sign.
(D) a and d are of opposite signs.

Progressions / Series
Directions for questions 1 and 2: Select the correct
alternative from the given choices.
1.

Find the number of common terms of the


sequences 24, 29, 34, .. 474 and 25, 29, 33,
, 485
(1) 20 (2) 21 (3) 25 (4) 24 (5) 23

2.

Nilgiris coffee cost `(110 + 0.2N) per kg on the


Nth day of 2006 where 1 N 200. Its price

th

remained constant that year from the 200 day.


Coorg coffee cost `(97 + 0.25N) per kg on the
Nth day of 2006 where 1 N 365. Find the date
in 2006 on which the prices of the two varieties of
coffee were equal.
(1) June 25
(2) August 10
(3) July 31
(4) August 20
(5) July 21

Directions for questions 3 and 4: Answer these


questions based on the given information
M and N are positive quantities. Let g1 = M and h1 = N.
When x is even, gx = Mhx 1 and hx = Mgx 1
When x is odd, gx = Nhx 1 and hx = Ngx 1
3.

9.

9
10. If S =
7
3
(A)
2

x 1

(3) M(MN)

(M + N)

1
x 1
2

126

ad infinitum, S =
124
5
(C)
(D) 3
3

1 3
5
7
9
27
and
+ +
+
+
+ ... +
2 6 12 20 30
210
1 1 1
1
+ + + ... +
, then find the value of
S2 =
2 3 4
14
2S2 S1
4
3
1
3
(A)
(B)
(C)
(D)
5
4
2
2

(1) M(MN) 2 (M + N)
(2) MN 2

28 65

26 63
1
(B)
2

11. If S1 =

Which of the following equals gx + hx when x is even?


1

Find the sum of the first 40 terms of the series


2 + 3 + 4 + 6 + 8 + 9 + 16 + 12 + . . .
(A) 621
(B) 221 + 628
20
(C) 220 + 628
(D) 6 + 629

12. If a, b, c, are 3 positive numbers in geometric


12
15
x
progression and a = b = c , then x =
(A) 18
(B) 20
(C) 21
(D) 24

(M + N)

(4) MM 2 (M + N)
1

(5) M(MN) 2
4.

x 2

13.

(M + N)

1
3
and N = , find the least odd x for which
4
4
gx + hx < 0.02?
(1) 3 (2) 5 (3) 7 (4) 9 (5) 11

If M =

Directions for questions 5 to 17: Select the correct


alternative from the given choices.
5.

If

1
+
10.11.12.13

14. If S=

2149
5616600
1429
(C)
5666100

1
+
11. 12. 13. 14

1
, then the value
99.100.101.102

6.

1249
5666100
4219
(D)
5616600

(B)

Find the sum of all the five


leave a remainder of either
by 8.
(A) 659100000
(B)
(C) 1318200000
(D)

digit numbers which


4 or 6 when divided
561900000
1237522500

7.

If the sum of the first n terms of an arithmetic


progression in 2400 and the sum of next n terms
as 7200, then find the ratio of first term and
common difference.
(A) 3 : 2
(B) 2 : 1
(C) 1 : 2
(D) 2 : 3

8.

If the nth term of a series is given by tn =


t
then 31 is
t 27
(A) 0
(C) 1

t n 1 + t n + 1
2

15. The nth term tn of a sequence is defined by tn = tn-1


tn-2. If t1 = 3, t2 = 4, and tn tm is a multiple of 10,
which of the following is not a possible value of
(m, n)?
(A) (10, 16)
(B) (16, 10)
(C) (7, 1)
(D) (8, 2)
16. The sum of n terms of an arithmetic progression,
starting with the 11th term is directly proportional
2
to n . The sum of 2 of the terms of the arithmetic
progression is 0. Which of the following cant be
either of these two terms?
th
(B) 11th
(C) 21st
(D) 16th
(A) 5
17.

1(32 ) + 2( 42 ) + 3(52 ) + .... + 10(122 )


=
3(12 ) + 4(22 ) + 5(32 ) + ... + 12(102 )

(A)
(B) 1
(D) None of these

1
1
1
1
1
1
1

+. . . +
1
2
3
4
5
6
99

1
, the value of S is also equal to
100
1
1
1
1
1
(A)
+
+
+
+... +
1
2
3
4
25
1
1
1
1
+
+
+ ... +
(B)
51
52
53
100
1
1
1
1
+
+
+ ...
(C)
50
51
52
100
(D) None of these

of k is
(A)

1
1
1
1
+
+
+ . . . +
is equal
1 .2 .3 2 .3 .4 3 .4 .5
13.14.15
to
59
26
1
31
(B)
(C)
(D)
(A)
105
105
7
210

4784
3795

(B)

15
11

(C)

29
23

(D)

6
5

Functions
Directions for questions 1 to 11: Select the correct
alternative from the given choices.
1.

G(x) is a function satisfying

G(a)
a
= G for all
G(b)
b

1
, find G(9).
9
1
(2)
(3) 81
9
(5) Cannot be determined

real a, b If G(3) =
1
81
(4) 9

(1)

2.

G(y) is a function satisfying the following conditions.


x

G(1) = 5400 and G(y) = X2G(X). where X is a

9. The continuous function f(x) satisfies


following conditions x
f(x + 2) f(x) = 4 for x 2
f(x + 2) + f(x) = 6 for 0 < x < 2
and f(x + 2) f(x) = 4 for x 0
also f(0) = 5
Which of the following is f(x)?
(A) 2x 4 + 1
(B) 2x + 5

(C) 4 x 2 + 3

3.

If f(xy) = f(x) + f(y) + f(x) f(y)


then the value of f(1/6) is
17
2
9
(B)
(C)
(A)
3
8
9

th

4.

5.

x 1 n
, f (x) = f1[fn-1 (x)], find f8 (x)
7x + 2
2x 1
2x + 1
(B)
(A)
7x + 1
7x 1
2x + 1
2x 1
(D)
(C)
7x 1
7x 1
1

11. If f (x) =

10
2
and f(6) =
,
3
9

(D)

17
8

Averages
Directions for questions 1 to 5: Select the correct
alternative from the given choices.
1.

Twelve years ago, the average of the ages of the


members of a joint family having ten members
was 25 years. Four years later a member aged
50 years died and a child was born in the family
that year. Four years after that, another member
aged 50 years died and another child was born.
Find the present average age of the members of
the family (in years).
(1) 26 (2) 27 (3) 28 (4) 29 (5) 30

2.

There are two groups (A & B) of children in a joint


family. There are 3 more children in B as
compared to A. While the average age of the
children in A is 6 years more than that of B, the
combined age of the children in A is less than
that of B. After 4 years, the difference in the
combined ages of group A and group B will
double. If there are less than 10 children in the
family, which of the following can be the average
age (in years) of the children in B?
(A) 9
(B) 8
(C) 7
(D) 5

3.

The number and the average weights of different


groups of children are given below:
Number
Average Weight (Kg.)
M
40
M+2
45
M+2
50
7
60
If the average weight of the entire class is 50 kg.
Find the total number of students.
(A) 20
(B) 21
(C) 22
(D) 23

x
, then the value of f(f(f(f(f(f( 3))))) =
If f(x) =
1+ x
3
5
4
6
(B)
(C)
(D)
(A)
26
25
37
19

If f(x +2) = f(x) + 7 and f(1) = 2; f(2) = 5 then the


ratio of f(150) to f(75) is
(A) 523 : 261
(B) 253 : 621
(C) 427 : 673
(D) None of these

6. If f (x + y) = f(x) + f(y) , where x 1, y 1 and f(7) =


24.5, then find the value of f(1) + f(2) + f(3) + f(4) +
f(5) + f(6) + f(7).
(A) 63
(B) 84
(C) 98
(D) None of these
7. If f(x+1) f(x) = ax + b, then which of the
following is true about the graph of y = f(x)?
(A) A line with a slope of a
(B) A line with a slope of b
(C) A parabola
(D) None of these
8.

Consider the function


p
q
r
s
f(x) = (x + 3) (x + 2) (x - 1) (x 3) , where p. Q.
R. s are nonnegative integers.
f(3.5) = a, f(2.5) = b, f(0.5) = c
f(1.5) = d, f(3.5) = e
b
Also, ab > 0,
<0, cd > 0 and d + e = d e .
c
Which of the following can be the value of
(p, q, r, s)?
(A) (2, 3, 5, 1)
(B) (4, 3, 2, 0)
(C) (3, 2, 5, 1)
(D) (0, 1, 2, 3)

(D) 2 x 4 + 1

10. If f(x + 1) f(x) = x and f(x) is an n degree


expression in x, then n =
(A) 2
(B) 3
(C) 4
(D) Cannot be determined

y =1

natural number greater than 1. Find G(6).


2100
2400
1800
(1)
(2)
(3)
7
7
7
1500
(5) 240
(4)
7

the

4. There are two vessels P and Q, P containing 120


L of milk and Q containing 120 L of water. In the
first operation 30 L is removed from P and poured

into Q and then 30 L from Q is poured back into


P. Like this, one more operation takes places i.e.
transferring 30 L from P to Q and then
transferring 30 L from Q to P. What is the ratio of
milk and water in P, finally?
(A) 2 : 1
(B) 13 : 6
(C) 15 : 7
(D) 17 :8
5. Dinku Beora was a chronic alcoholic and
because of persistent, health problems he
decided to quit drinking. He devised an
ingenuous way of doing so. He bought a 750 ml
goodbye bottle of Old Monk. On the first day, he
drank 5% of the contents in it and replaced that
quantity with water. Next day he drank 10% of the
contents in the bottle and replaced it with water.
th
Like this he continued on the 19 day he drank
95% of the contents in the bottle and replaced it
with water and on the 20th and last day he drank
the entire contents in the bottle. Find the ratio of
the total quantity of alcohol and water that he
drank in the entire process.
(A) 2 : 21
(B) 2 : 19
(C) 3 : 28
(D) None of these

Quant Based Reasoning


Directions for questions 1 and 2: Answer these
questions based on the following information. P, Q, R,
S and T are 5 horses. They participated in a race.
The following are the rules of the race.
(1) A person who bets on the winning horse gets
4 times the bet amount.
(2) A person who bets on the horse coming second
gets 3 times of the bet amount.
(3) A person who bets on the horse coming in third
gets back his amount.
(4) Other persons lose their amount.
Mohan had placed his bets on Q, R and T.
The amounts he bet on Q, R and T were `2000,
`4000 and `6000 respectively. He ended up with no
gain and no loss.
1.

2.

Which of the following is not possible?


(1) At least two horses finished before P.
(2) There were three horses between Q and P.
(3) T finished last.
(4) S came in second.
(5) There were three horses between T and R.
Suppose S finished in the fourth position. Then
which of the following is not possible?
(1) P finished first.
(2) One horse finished between Q and R.
(3) Q came in second
(4) T finished last.
(5) R came second.

(1) if the question can be answered from A alone but


not from B alone.
(2) If the question can be answered from B alone but
not from A alone.
(3) if the question can be answered from A alone as
well as from B alone.
(4) if the question can be answered from A and B
together but not from any of them alone.
(5) if the question cannot be answered even from A
and B together.
A certain number of players participated in a
tournament, played according to the following
rules. The number of players at any stage is
denoted as N.
N
pairs.
(i) if N is even, the players are grouped into
2
The players in every pair play against each other.
The resulting winners move on to the next round.
(ii) If N is odd one player is allowed to move on to
the next round. He is said to be given a bye. The
N1
remaining N 1 players are grouped in to
2
pairs who play against each other. The resulting
winners move on to the next round. The players
who lose are eliminated from the tournament.
From the rules above, it follows that if there are
N
players move on to
N players in a round, then
2
N+1
players
the next round if N is even and
2
move on to the next round if N is odd.
This process continues until the final round,
which is played between two players. The winner
in this round is the champion.
1.

Find the number of matches played by the champion.


A. In the first round, there were 169 players.
B. The champion was given a bye only once.

2.

The number of players in the first round was M


where 129 < M < 256. Find M.
A. One player received a bye while moving from
the third to the fourth round.
B. Only one player received a bye in the entire
tournament.

Directions for questions 3 to 6: Each question is


followed by two statements A and B. Indicate your
responses based on the following directives:
Mark (1)
Mark (2)
Mark (3)

Mark (4)

Data Sufficiency
Directions for questions 1 and 2: Answer these
questions based on the instruction below.
In the questions below, each question has two
statements A and B following it. Mark your answer as

3.

if the question can be answered using A


alone but not using B alone.
if the question can be answered using B
alone but not using A alone.
if the question can be answered using A
and B together, but not using either A or
B alone.
if the question cannot be answered even
by using A and B together.

Class X has 80 students. The average height of


the students in it is 140 cm. It has two sections,
A and B, with equal number of students in each
section. The average height of A exceeds that of
B. Mohan is the tallest in A and Sohan is the

10

shortest in B. If each of these students is


transferred to the other section, the average
heights of the sections would get interchanged.
Find the height of Mohan.
(A) The average heights of A and B differ by 2 cm.
(B) If Sohan shifted from B to A, the average
heights of the sections would become equal.
4.

A company has to store at least 270 kilolitres of


water at all times to meet safety and regulatory
requirements. It is considering having a spherical
tank whose wall thickness is uniform and whose
outer radius is 6 meters for this purpose. Will the
tank meet the company requirements?
(A) When empty, the tank weighs 36000 kg. It is
made of a material whose density is 4 gm/cc.
(B) The tanks inner radius is at least 4.5 metres.

Directions for questions 3 to 8: Select the correct


alternative from the given choices.
3.

The price of coffee was increased by 40% but Raj


was willing to increase his expenditure by 12%.
Find by what percentage should he decrease his
consumption.
(A) 10%
(B) 15%
(C) 20%
(D) 25%

4.

The ratio of the populations of cities X, Y and Z in


2008 was 3 : 5 : 6. The percentage increases in
the populations of X, Y and Z from 2008 to 2009
were 10%, 12.5% and 15% respectively. Find the
percentage increase in their total population from
2008 to 2009.
(A) 12%
(B) 14%
(C) 13%
(D) None of these

5.

P, Q and R are three integers. Find the maximum


value of PQ + QR + PR.
(A) P = Q R
(B) P + Q + R = 84

5.

If the cost of a ball pen reduces by 20%, Raj can


buy 90 more ball pens for `3600. Find the cost (in
`) of a ball pen.
(A) 12.5
(B) 10
(C) 15
(D) 20

6.

P is a point on AB. Rohit wanted to draw a


square ABCD but failed to do so. Why did he fail?
(A) PC = 3 cm
PD
(B) PC =
3

6.

Ashok made a loss of 15% by selling 96 apples


for `2040. How many apples must he sell for
`2600 to make a 30% gain?
(A) 80
(B) 100
(C) 65
(D) 104

7.

Three persons A, B and C have their monthly


incomes in the ratio 6 : 7 : 8. Their monthly
expenditures are in the ratio 5 : 6 : 10. The monthly
savings of C is 37.5% of his monthly income. Find
what percent of Bs savings was As savings?
(A) 50%
(B) 662/3% (C) 831/5% (D) 87.5%

Percentage, Profit & Loss


Directions for questions 1 and 2: Answer these
questions based on the information below.
Mohan was considering three alternatives for investing
a certain amount. He wanted to get the maximum
possible assured return on his investment. The three
alternative are given below. He could make use of
each completely or partially along with the others.
Alternative 1: Invest in the mutual funds of PQR Ltd.
A rise in the stock market will result in a return of 8%
and a fall will result in a return of 10%.
Alternative 2: Invest in the mutual funds of RQP Ltd.
A rise in the stock market will result in a return of 5%
and a fall will result in return of 4%.
Alternative 3: Invest in a public sector bank which
promises a 0.4% return.
1.

2.

Find the greatest assured return for Mohan.


2
%
(1) 0.4%
(2) 0.5%
(3)
3
5
%
(4) 0.8%
(5)
6
Find the strategy which will maximize
guaranteed return to Mohan.
(1) 100% in alternative 3.
(2) Equal investment in each alternative.
(3) Investments in alternatives 1 and 2 in
ratio 1 : 2
(4) Investments in alternatives 1 and 2 in
ratio 2 : 1
(5) Investments in alternatives 1, 2 and 3 in
ratio 2 : 3 : 4

the

the
the
the

8. Pradeep bought a puppy for a certain price.


He sold it to his neighbour at a profit percent
whose magnitude was equal to the profit realised
by him in the transaction. But after a couple of
days the neighbour sold it back to Pradeep at
20% loss. Effectively, 27.5% of the cost price was
refunded to Pradeep. At what profit percentage
did he sell the puppy to his neighbour?
(A) 30%
(B) 37.5% (C) 27.5% (D) 32.50%

Simple Interest Compound Interest


Directions for questions 1 and 2: Select the correct
alternative from the given choices.
1.

The difference between the simple interest and


the compound interest for two years on a sum
invested at 16% p.a. is `384. Find the sum
(in `)
(A) 13500 (B) 15000 (C) 14250 (D) 12750

2.

Ramu took a certain loan at Simple Interest in


2000 for a period of 4 years. The rate of interest
was constant throughout the loan period
whereas had he cleared the loan after 9 years,
he would have paid `90,000. Where as had he
cleared the loan after 12 years, he would have
paid `105,000. Find the amount (in `) he paid to
clear the loan.
(A) 45000 (B) 65000 (C) 70000 (D) 75000

11

them working each day, the total time taken


would be 241/2 days. Find the number of days A
alone would take to complete the work if B is
faster than A.
(A) 28
(B) 21
(C) 24
(D) 25

Time and Work


Directions for questions 1 to 7: Select the correct
alternative from the given choices.
1.

Pipe X can fill a tank in a certain time. It was


opened at 12 pm. Due to a leak at the bottom of
the tank, the tank was filled only at 12:50 pm.
If the leak can empty the tank in 200 minutes,
then find the time (in minutes) in which X can fill
the tank.
(A) 30
(B) 45
(C) 40
(D) 50

2.

8 men can build a wall, 8 m long in 8 days,


working 8 hours a day. Find the number of
hours per day for which 16 men are required to
work, to build a wall 16 m long in 16 days.
(A) 2
(B) 8
(C) 16
(D) 4

3.

50 men can complete a job in 20 days working


9 hours a day. They started the job. They
worked 9 hours each day for the first x days.
At the end of x days, 5 men left. The remaining
job was completed by the remaining men
working 8 hours a day in 12.5 days. Find x.
(A) 12.5 (B) 8
(C) 9
(D) 10

4. If A, B, C, D work independently, the amount that


A is paid for 3 days is equal to that paid to B for
4 days. The amount paid to C for 3 days is equal
to that paid to D for 2 days. B and D are paid
equal amounts for equal duration. A and B
together complete a piece of work for which they
are paid `1680. If all 4 had completed the same
work together, what would A's share have been
(in rupees)?
Assume that equal amounts are paid for equal
work.
(A) 960
(B) 800
(C) 560
(D) 630
5. For workers working on a construction site, the
rate of doing work for men and women increases
in winter with respect to that in summer by
1
33 % and 50% respectively. 4 men and 12
3
women can complete a certain piece of work in
120 days in summer. The time taken triples if the
women do not turn up for the work. If a men and
b women can complete two times the given work
in 180 days in winter then find the possible
number of ordered pairs of (a, b).
(A) 2
(B) 20
(C) 10
(D) 5
6. A and B are two daily labourers who work on a
maintenance site. The daily wage of A is 40%
less than the daily wage of B. A and B together
worked on a certain project and completed it in
72 days. As a result, at the end of the project they
together received a certain amount. If that entire
amount were to be earned by A alone, then for
how many days would he need to work ?
(A) 144
(B) 216
(C) 288
(D) 192
7. A and B can together do a piece of work in
12 days. If A completed half the work and the
other half is completed by B with only one of

Venn diagrams
Directions for questions 1 to 3: Select the correct
alternative from the given choices.
1.

In a class, 30% of the students like tea and 40%


of the students like coffee. 20% of the students
who like tea also like coffee. Find the
percentage of the studies who like neither tea
nor coffee.
(A) 32% (B) 34%
(C) 38%
(D) 36%

2.

In a locality, 180 residents watch only Sony TV,


210 residents watch only Star Plus and 150
residents watch only Zee TV. 540 residents
watch atleast one of Sony TV and Star Plus. At
most 340 residents watch Star Plus. 90
residents watch all the three channels. Find the
minimum possible number of students who
watch Sony TV and Zee TV but not Star Plus.
(A) 40
(B) 50
(C) 20
(D) 30

3.

In a group, 60% of the boys and 50% of the girls


like cricket. 45% of the boys and 55% of the
girls like volleyball. Number of students who like
cricket is 7 more than that of students who like
volleyball. The difference of the number of boys
and girls who like cricket and that of the number
of boys and girls who like volleyball are in the
ratio 16 : 5. Find the strength of the group.
(A) 90
(B) 100
(C) 110
(D) 120

Trigonometry
Directions for questions 1 to 3: Select the correct
alternative from the given choices.
1. T1 and T2 are two towers and Raju was on the top
of the tower T1. He realized that there were two
points on the ground such that the angle of
elevation of T1s top from each of those points
was . The distances from, T2s bottom to the top
of T1 as well as to each of the points was
30 feet. The area of the triangle formed by the top
of T1 and each of the points can be (in feet).
(T2 's bottom and the two points on the ground
are collinear)
(A) 225
(B) 960
(C) Both (A) and (B)
(D) Neither (A) nor (B)
2. A ladder has a length of 10 m. It makes an angle
of 45 with a wall. It touches the wall at a point P.
There are two points on the ground. The angle of
elevation of P from each of these points is 60.
Find the distance between these points (in m).
(The two points on the ground and the bottom of
the wall are collinear)

(A)

7 6
3

(B)

10 6
3

(C)

8 6
3

(D)

4 6
3

12

pq + qr + rp
, where p, q, r are
p 2 + q2 + r 2
real numbers such that the sum of any two
exceeds the third, which of the following is not a
possible value of ?
(A) 20 (B) 40
(C) 50
(D) 80

3. Given that cos =

Indices, Logarithms, Surds


Directions for questions 1 to 7: Select the correct
alternative from the given choices.
1.

33x + 2 92x 1 = 118098, find the value of 6x + x6.


(A) 945
(B) 559
(C) 2403
(D) None of these

2.

Find the value of


1
17 16 + 16 17

(A)

3.

20
15

(B)

1
18 17 + 17 18

7
44

(C)

+ ... +

1
121 120 + 120 121

37
44

(D)

13
15

1
1
6
= 3, find the value of x + 6 .
x
x
(A) 729
(B) 326
(C) 322
(D) 324

If x +

4. If a = 5 2 6 and b = 5 + 2 6 , evaluate
4

a +b

(A)
5.

4901
49

(B)

4799
485

(C) 10

(D)

4801
485

If log2log4x = log4log2x, find x.


(A) 16

4. If x1, x2, x3, x4, x5 and x6 are positive and x1 x2 x3


x4 x5 x6 = 1; then the minimum value of (x1 + 4)
(x2 + 4) (x3 +4) (x4 +4) (x5 +4) (x6 +4) is
(A) 15625
(B) 1425
(C) 13225
(D) 11025
1
1 1 1 1
, then S lies in
+ + + + ... +
512
1 2 3 4
which of the following ranges?
(A) (4, 5.5)
(B) (5.5, 9)
(C) (9, 10.5)
(D) (10.5, 12)

5. If S =

6. P is any positive number such that it is possible


to find 4 positive numbers whose product is P
and whose sum S P. Among the numbers 4, 5,
6, 7, how many are possible values of P?
(A) 0
(B) 1
(C) 2
(D) 3
7. If x3 + y3 + z3 = 125, a2 + b2 + c2 = 16, ax + by + cz =
20 and x, y, z, a, b, c are integers, then which of
the following cannot be the value of
a+b+c
?
4 x 2 + y 2 + z2

1
(A)
25

a3 + b3

3. What is the set of values of x for which the


4x 2 9x + 8
inequation 2
> 3 is satisfied
x 5x 6
(B) (, 6) (1, )
(A) (6, 1) (1,)
(C) (, )
(D) (, 1) (6, )

(B) 8

(C) 8 2

(B)

1
27

(C)

1
33

(D)

1
49

8. How many integral values of x satisfy the


following inequality
14 < x 5 + x + 3 + x + 7 < 25?

(A) 9

(B) 11

(C) 13

(D) 16

(D) 4 2

6. If 1, log7(4x + 5), log7(4x + 1 1) are in arithmetic


progression, which of the following is a possible
value of x?
(A) 1
(B) 1.5
(C) 2
(D) 2.5
7. Let t = 3(log3a)2 9log3a + 9. Which of the
following is true about the equation at=27?
(A) It has exactly one distinct solution for t.
(B) It has exactly two distinct solutions for t.
(C) It has exactly three distinct solutions for t.
(D) It has no real solutions for t.

Modulus / Inequalities
Directions for questions 1 to 10: Select the correct
alternative from the given choices.
1. How many integer values of x exist that do not satisfy
the inequality (x2 + 4x 32) (x2 + 2x 8) > 0?
(A) 7
(B) 8
(C) 6
(D) 4
2. Under which of the following conditions the
a2 b2

a b is true?
inequation
b
a
(B) a b and ab > 0
(A) a b and ab < 0
(C) Either (a) or (b)
(D) None of these

9. Find the area described by the inequality


x + y + xy 2

(A) 3

(B) 2

(C) 2.5

(D) 4

10. In triangle PQR, p, q and r are the lengths of the


sides opposite P, Q, and R respectively. If (p + q
+ r)2 = 3(pq + qr + rp), what can be said about the
triangle PQR?
(A) It is equilateral.
(B) It is isosceles.
(C) It is scalene.
(D) Cannot be uniquely determined

Operator Based Questions


Directions for questions 1 to 3: Select the correct
alternative from the given choices.
1. Given that a b = 4a + 3b + 7ab and a a < b
b, then which of the following is true?
(A) (a b) (a + b + 1) < 0
(B) (a + b) (a b + 1) > 0
(C) (a b) (a + b+ 1) > 0
(D) (a+ b) (a b + 1) > 0

13

2. If x @ y = y 4 x + + x
x

1
of 6 @
is
6
3891
(A)
36
4179
(C)
36

3.

(B) 1
(D)

1297
36

The operations * and are defined as


a * b = a + b + ab and a b = ab (a + b).
Find the value of (3 * 4) (4 5).
(A) 19 (29) (113)
(B) (85) (30)
(C) 19 (180) (199)
(D) 84 (29) (113)

Numbers
Directions for questions 1 to 43: Select the correct
alternative from the given choices.
1. Raju wrote the first 50 natural numbers one after
another on a black board. He then carried out the
following procedure 49 times. In each instance,
he erased two numbers, say p and q and
replaced them by a single number p + q 1.
Find the final number left on the board.
(1) 1224
(2) 1275
(3) 1276
(4) 1274
(5) 1226
2.

Find the last two digits of 73024.


(1) 41
(2) 81
(4) 01
(5) 61

(3) 21

3. N is a natural number. Div(N), a function of N, is


defined as follows:
Div(N) = N if N 9
= Div(S(N)), otherwise,
where S(N) is the sum of the digits of N.
For instance, Div(8) = 8,
Div(625) = Div(6 + 2 + 5) = Div(13) = Div(1 + 3)
= Div (4) = 4 etc.

Find the number of positive integer values of N


less than 600, for which Div (N) = 9.
(1) 44
(2) 55
(3) 66
(4) 77
(5) 67
4.

Find the value of


upto 2005 terms
1
(1) 2005
2006
1
(3) 2006
2005
1
(5) 2006
2007

1+

of
the
PQ=
(1) 2
(4) N

3
4 y + , then the value
y

resulting

even

elements,

then

(3) 1

(2) 1
(5) N1

6. X and Y are natural numbers. X is odd and less


than 100. Find the number of solutions of
1
1 3
=
.
X 18 Y
(1) 4 (2) 3 (3) 2 (4) 1 (5) 5
7. A tournament had 2N + 1 teams t1, t2, .. t2N+1
where N > 6. Each team had x players where
x > 4. The following pairs of teams have a
common player : t1 and t2N+1, t2 and t2N, .. tN and
tN+1. These are the only pairs of teams who have
a common player. Find the total number of
players in the 2N + 1 teams.
(1) N(x 1) + x
(2) N(2x 1) + x
(3) N(2x 2) + x
(4) N(x + 1) + x
(5) N(x + 2) + x
8. A four digit number has the form AABB. It is also
a perfect square. How many (A, B) are possible?
(1) 0 (2) 3 (3) 4 (4) 1 (5) 2
9.

Let M = 3(3!) + 4(4!) + .. + 15(15!)


What is the remainder when M 15 is divided by
14! 2?
(A) 14! 443
(B) 14! 459
(C) 459
(D) 443

10. A number when divided by 5, 6, 7 and 8 leaves


remainders of 3, 4, 5 and 6 respectively.
How many such 4-digit numbers are there?
(A) 11
(B) 10
(C) 9
(D) 12
11. If x =
1
32

(A)

1
1
1
+ 2 + 2 + .... , what is the value of
22
3
4
1
52

1
72

+ .... in terms of x?

3x 1
3x
x 1
(B)
1 (C)
4
4
2

12. If k =

49 80 42 80
49 79 + 42 79

(D)

x
1
2

, then

(A) 0 < k 1
(C) 4 < k 7

(B) 1 < k 4
(D) k > 7

1
1
1
1
+
+ 1+ 2 + 2 +
12 22
2
3

13. If A = 71421 . 98 105 112 ...189 196,


what is the remainder when A is divided by 9?
(A) 1
(B) 3
(C) 7
(D) 5

1
2006
1
(4) 2005
2005

14. If 2a + 5b = 7(a2b5)1/7 and x =

(2) 2006

5. Set A = {3, 4, .. 2N + 1, 2N + 2}, where N is a


natural number. Each odd element in it was
increased by 3 and each even element in it was
increased by 1. P denotes the average of the
resulting odd elements and Q denotes the average

a + b
, what is
2

the value of 2(x + a)2 + 5(x b)5?


(A) 7
(B) 168
(C) 3
(D) None of these

15. Which of the following is a rational number?

(A) (7 + 4 3 )50 + (7 4 3 )50


(B) (7 4 3 )50 + (7 + 4 3 )50
log

(C) 4 3 4 3 + 7 log7
(D) None of these

4 3

14

16. Which of the following is prime?


(B) 296 + 1
(A) 270 + 1
160
(C) 2 + 1
(D) None of these
17. A 200 page book is formed by using 50 sheets,
folded in the middle and stapled along the fold,
with each sheet providing 4 pages. The pages
are then numbered from 1 to 200. The nth sheet is
removed from the book. What is the sum of the
pages on this sheet?
(A) 400 2n
(B) 400 n
(C) 200 + 4n
(D) None of these
18. If u + v + w + x + y = 15, what is the maximum
value of uvx + uvy + uwx + uwy?
(A) 125 (B) 144
(C) 3125 (D) 243
10

19. The remainder when 1301 1301 is divided by


21 is
(A) 1
(B) 2
(C) 19
(D) 20
20. If a, b c, d are natural numbers such that
23 < a < b < 40 < c < d < 50, how many values
are possible for the quadruplet (a, b, c, d)?
(A) 4284 (B) 4320 (C) 4200 (D) 4165
21. A number N leaves a remainder of 7, 10 and
17 respectively when divided by 15, 21, and 35.
What is the remainder when N is divided by 105?
(A) 73
(B) 59
(C) 52
(D) 55
22. How many five digit numbers with digits that are
not necessarily distinct are divisible by 6 but not
by 12?
(A) 7500 (B) 7499 (C) 14999 (D) 15000
23. Which of the following numbers does not divide
(412 1)?
(A) 5
(B) 63
(C) 255
(D) 127
24. Find the number of factors of 243243 which are
multiples of 21.
(A) 20
(B) 21
(C) 22
(D) 24
25. What is the remainder when 4911 + 5011 + 5111 +
5211 is divided by 202?
(A) 0
(B) 101
(C) 201
(D) 1
26. Q is the number formed by knocking off all the
terminal zeros from 256!. What is the index of the
highest power of 12 that divides Q?
(A) 126
(B) 97
(C) 96
(D) None of these
27. Let N = (31)(32)(33)(98)(99)(100)
If N is divisible by 12n where n is a natural
number, find the maximum value that 'n' can take.
(A) 30
(B) 34
(C) 35
(D) 40
28. 'a' and 'b' are prime numbers and n is an integer
such that 1 n ab. What is the sum of all the
possible values of n such that n and ab are
coprime to each other?
ab a b + 1
(A) (a 1) (b 1)
(B)
2
ab(a 1)(b 1)
(C) ab
(D)
2

29. Set A = {1, 4, 7, 10, 13, , 100}. B is a non


empty proper subset of A such that all the
elements of B are divisible by 7. Find the number
of such subsets.
(A) 5
(B) 31
(C) 32
(D) 63
30. There are 4 distinct integers p, q, r and s, such
that the equation (n - p) (n - q) (n r) (n s) = 9
is satisfied for some integral values of n. How
many integral values of n satisfy the given
equation for a particular set of values of p, q, r,
and s?
(A) 0
(B) 1
(C) 2
(D) More than 2

(3333344444

31. Find the remainder when


4444433333) is divided by 7
(A) 0
(B) 2
(C) 3

(D) 6

32. Find the remainder when 4 + 44 + 444+ 4444+ ...


+ 444 . . . (50 digits) is divided by 9.
(A) 0
(B) 3
(C) 6
(D) 8
33. Consider the equation

1 1 1
+ = , where a and b
a b 6

are integers. How many ordered pairs (a, b) exist


which satisfy the given relation?
(A) 9
(B) 10
(C) 17
(D) 18
34. The difference between the squares of two
integers is 420. How many such pairs of integers
exist?
(A) 4
(B) 8
(C) 12
(D) 16
35. How many 9 digit multiples of 6 can be formed
using only the digits 8 or 9?
(A) 56
(B) 84
(C) 86
(D) None of these.
5

36. (a + b + c + d) (a + b + c + d ) is always
divisible by
(A) 24
(B) a + b + c + d
(C) 9
(D) 5
37. If A = 1! + 2! + 3! + 4! + .......+ 49! and
B = 1! + 2! + 3! + 4! +......+168!, then which of the
following is true?
(A) A is a perfect square.
(B) (B) is a perfect square.
(C) Both A and B are perfect square.
(D) Neither A nor B is a perfect square.
38. What is the sum of the even factors of 3600?
(A) 4030
(B) 12896
(C) 6046
(D) 12090
39. Consider all 5-digit numbers for which the sum of
the digits is 41. How many of these are divisible by
11?
(A) 10
(B) 12
(C) 16
(D) More than 16
40. In how many ways is it possible to express 36 as
a product of 3 positive integers?
(A) 8
(B) 6
(C) 12
(D) None of these

15

41. W is a whole number. The function root(W) is


defined as follows
root (W) = W if W 2
= root (W 3) if 3 W 9
= root (sum (W)) otherwise, where sum (W)
denotes the sum of the digits of W
For example
Root (7) = root (7 3) = root (4) = root (4 3)
= root (1) = 1
Root (245) = root (2 + 4 + 5) = root (11)
= root (1 + 1)= root (2) = 2
How many values of W less than 500 satisfy the
condition root (w) = 0?
(A) 165
(B) 166
(C) 167
(D) 168
42. The 18-digit number X has a 7 in the units place.
If this 7 is transposed to the front of the number,
the resulting 18-digit number is 5 X. Find X.
(A) 124587124587124587
(B) 124857124857124857
(C) 142857142857142857
(D) 142587142587142587
43. How many numbers less than or equal to 1108
are coprime to 252?
(A) 311
(B) 312
(C) 316
(D) 318

Directions for questions 44 to 46: These questions


are based on the following data.
Professor Calculus devised a game to pass his time.
The game was played in the following manner. He
kept 500 coins numbered from 1 to 500, on a table in
sequence all showing heads.
In round 1, he flipped all the coins such that each
showed a tail. In round 2, he flipped only those coins
whose position was a number divisible by 2.
In round 3, he flipped only those coins whose position
was a number divisible by 3 and so on.
He continued flipping the coins for 500 rounds.
44. What was the greatest number of consecutive
coins all showing heads at the end of the game?
(A) 16
(B) 41
(C) 42
(D) 43
45. Among the following coins, which coin was
flipped the greatest number of times?
(A) 210
(B) 324
(C) 288
(D) 240
46. How many coins did Professor Calculus flip in
round 13, which were flipped only once in the
previous rounds?
(A) 7
(B) 8
(C) 11
(D) 38

Directions for questions 47 to 49: Select the


correct alternative from the given choices.
47. If the sum of all integers between 3n and
3n+3(both exclusive) is divisible by 70 where n is a
positive integer, then which of the following is
necessarily true about n?
(A) It is divisible by 4
(B) It is a multiple of 6
(C) It is divisible by 7
(D) It is divisible by 5.

48. Find the units digit in 1 + 2 + 3 + +90 .


(A) 0
(B) 5
(C) 6
(D) 1
49. If S(N) denotes the sum of the digits of N, then
what is the remainder when S(1) + S(2) + S(3) +
..+ S(99) is divided by 99?
(A) 0
(B) 9
(C) 18
(D) 36

Probability
Directions for questions 1 to 4: Select the correct
alternative from the given choices.
1. There are 50 tokens numbered 1 to 50 kept on a
table. You are asked to pick two tokens at
random from the table one after the other. What
is the probability that the difference between the
numbers on the tokens picked by you lies
between 1 and 6 (both excluded)?
36
186
(A)
(B)
245
1225
372
186
(D)
(C)
2450
1225
2. A number is selected at random from 1 to 105.
Consider the following 3 events.
P: X is coprime to 3`
Q: X is coprime to 5
R: X is coprime to 7
Which of the following is true about the events
P, Q, R?
(A) They are pairwise independent and mutually
exclusive.
(B) The are pairwise independent but not
mutually exclusive.
(C) They are not pairwise independent but are
mutually exclusive.
(D) They are neither pairwise independent nor
mutually exclusive.
3. Alsi Arora worked in a private company where the
working hours were from 9:00 am to 6 pm. In a
year on 25% of the days he arrived late to office
whereas on 35% of the days he left early from
office. If p is the probability of the number of days
that he worked for the entire working day, then
which of the following best describes the possible
values of p?
(A) P = 0.4875
(B) 0.25 P 0.65
.
(C) 0.25 P 0.65
(D) 0.40 P 0.65
4. Ramesh doesn't always have time to read the
daily newspaper and whenever he reads it he
does it either in the morning or in the evening.
The probability that he reads it in the morning is
10% and the probability that he reads it in the
evening is 30%. Find the expected number of
days in a month of 30 days on which he doesn't
read the paper at all.
(A) 16
(B) 18.9
(C) 21
(D) 20.5

16

Miscellaneous

Inequations

Directions for question 1: Select the correct


alternative from the given choices.

Directions for question 1: Select the correct


alternative from the given choices.

1. There is a sequence of 7 sets A1, A2, A4, each


consisting of 6 elements and another sequence of
n sets B1, B2, ,,,, Bn each consisting of 3 elements.
The union of all the A's is equal to the union of all
the B's. If each element of this union occurs in
exactly 3 of the A sets and 6 of the B sets, then
find n.
(A) 28
(B) 21
(C) 14
(D) Cannot be determined

Coordinate Geometry
Directions for question 1: Select the correct
alternative from the given choices.
1.

If the equation 9x2 y2 + 6y = A where A is a


constant, represents a pair of straight lines, then
which of the following gives the point of
intersection of those two lines?
(A) (1, 2)
(B) (0, 3)
(C) (0, 3)
(D) cannot be determined

1. A triangle with sides x, y and z is such that


x3 + y3 +z3 = 3xyz. Which of the following is true
regarding the triangle?
(A) It is an isosceles triangle
(B) It is an equilateral triangle
(C) It is an obtuse angled triangle
(D) It is a right angle triangle

Statistics
Directions for question 1: Select the correct
alternative from the given choices.
1. The median of 11 numbers is x. If the greatest
number is removed the median of the remaining
numbers is 10.5. If the smallest is removed the
median of the remaining numbers is 13. If x (or
one of the numbers which is equal to x) is
removed the median would be
(A) 11
(B) 11.5
(C) 12
(D) Cannot be determined

17

SOLUTIONS FOR QUANT REPLICA QUESTIONS THAT HAVE


APPEARED IN CAT IN THE LAST 4 YEARS
Permutations and Combinations

N2 5N + 6
will be non-mates of the
2
member (a, b)
Choice (4)

Now all these


Solutions for question 1:
1.

Least element of I = 1000.


Greatest element of I = 5000.
Let the required number be N.
N = Number of integers from 1000 to 5000 which can
be formed with 0 or 1 or 2 or 3 or 5.
If the integers are less than 5000, the thousands digit has
three possibilities (1 or 2 or 3). Each of the remaining
digits has five possibilities 0, 1, 2, 3 or 5
A total of (3) (5) (5) (5) or 375 integers can be formed
less than 5000.
The only integer not less than 5000 is 5000 itself.
N = 375 + 1 = 376
Choice (1)

6.

Let (a, b) and (b, c) be two mates in X


They have element b in common.
Now all the members which have b as one of the
elements will be a common mate to both (a, b) and
(b, c). Other than a, b and c, there are. N 3 elements.
Now with each of these n 3 elements with element b
can form a common mate.
In addition the member (a, c) will also be a common mate
to both the members (a, b) and (b, c).
So, total number of common mates = N 3 + 1
= N 2.
Choice (1)

Solutions for questions 2 and 3:

Solutions for questions 7 to 19:

Any shortest path from H to O must include AB.


The number of shortest paths from H to O is mn, where m
is the number of shortest paths from H to A and n is the
number of shortest paths from B to O
m = 4C2 = 6 and n = 7C2 = 21.
mn = 126.

7.

If the sum of the digits of a number is divisible by 9,


then the number is divisible by 9.
Sum of the given digits is 0 + 1 + 2 + 4 + 6 + 8 + 9 = 30.
We leave two digits such that the sum of the other five
digits is multiple of 9.
0, 4, 6, 8, 9 or 0, 1, 2, 6, 9 the number of arrangements
in each case 4! 41 hence 2.4.4! = 192.
Choice (B)

8.

If the last two digits are divisible by 4, then the number


is divisible by 4.
Given digits is 0, 2, 4, 5, 6, 7.
The possible two digits are 04, 20, 40, 60, 24, 52, 56,
64, 72, 76
The number of five digit number formed which are
end with 04 is _ _ _ 04 = 4P3.
The number of five digit numbers formed which are
4
end with 04 or 20, 40 or 60 = 4 P3 = 96.
The number of five digit number formed which are end
with 24 is _ _ _ 24 = 3 3 2 = 18
The number of five digit number formed which are
end with 24, 52, 56, 64, 72 or 76 = 6 18 = 108.
Hence the total number of five digit number which
are divisible by 4 can be formed using the given digits
= 96 + 108 = 204
Choice (C)

9.

All players who play the same game are treated as one
unit. Then four units can be arranged in a row in 4!
ways.
Now the players in each game can be arranged among
themselves in 6! 7! 8!and 9! ways respectively.
The required number of arrangements = 4!6!7!8!9!
Choice (D)

H
A

R
B

O
c

13
6

7
6
5
4
3
2
1

1
1

1
1
1
1
O
In the figure above, the number at a node indicates the
number of shortest paths to reach that node from O.
P

2.

mn = 126

Choice (4)

3.

There are 126 shortest paths from H to O and


13 shortest paths from O to C.
There are a total of (126) (13) or 1638 shortest paths
from H to C via O.
Choice (1)

Solutions for question 4:


4.

The terms in the expansion of (p + q + r)15 have the


x y z
form Kp q r where x + y + z = 15. The number of terms
is the number of non-negative integral solutions of
x + y + z = 15, which is equal to the number of positive
integral solutions of x + y + z = 18, which is 17C2 or 136.
Choice (1)

Solutions for questions 5 and 6:


5.

Let (a, b) be a member of X.


Now non mates of this member will have any of the
remaining elements other than a and b.
Remaining elements in N other than a and be is N 2
elements = N 2 C2 =

N2 = 5N + 6
(N 2) (N 3)
=
2
2 .1

10. Each student can be sent into any of the five sections.
Seven students can be sent 5 5 5 5 5 5 5
i.e 57 ways.
Choice (B)
11. The sum of the all the five digit numbers formed by the
digits a, b, c, d, e without repetition in 4! (a + b + c + d
+ e) (11111).
The sum of the all the five digit number formed by the
digits 2, 4, 6, 7, 8 is 4! (2 + 4 + 6 + 7 + 8) (11111)
= 7199928.
Choice (B)
12. Since 40 lines are given, The number of points of
intersection of these 40 lines is 40C2. But 2 lines are
concurrent they are intersected at only one point.
Similarly, 15 lines are concurrent they intersect at only
one point. 13 lines are parallel.
They do not intersect.
Hence required number of points of intersections
40
= C2 66 105 78 + 2 = 533.
Choice (C)

100

13. Let E = (x + y z)
n
a b c
The general term in the expansion is C1 x y z , where
a + b + c = 100.
For the terms which are negative c is odd, i.e., c = 1, 3,
5, 99.
a + b = 99, 97, --- 1 -------(A)
The number of negative terms is equal to the sum of the
number of non-negative integral solutions of the fifty
equations denoted by (A) which is 100C1 + 98C1 + . 2C1
= 100 + 98 + .. + 2
(50) (51)
Choice (C)
= 2
= 2550.
2
14. In the word EQUATION the vowels are A, E, I, O and V
whereas the consonants are N, Q, and T. Since no two
vowels or no two consonants are to be in the
alphabetical order, the vowels and the consonants must
occur in the reverse alphabetical order.
Vowels U, O, I, E, A consonants T, Q, N
The total number of arrangements possible with the 8
letters is 8! Among these the vowels can be arranged
among themselves in 5! ways, of which only one
sequence is permissible.
Similarly, the consonants can be arranged among
themselves in 3! ways, out of which only one sequence
is permissible.
Therefore, the number of such words =

1
1
1
1

)
2
6
24
120
60 20 + 5 1
) = 44
= 120 (
120
= 120 (

Therefore the total number of ways in which all the


parcels can be addressed = 9 (44) = 396.
Choice (A)
19. The number of triangles that can be formed is
12
3
4
5
C3 C3 C3 C3
= 220 (1 + 4 + 10) = 205
Choice (D)
Geometry / Mensuration
Solutions for questions 1 to 40:
1.

1
(PS (QR)
2
(PQ) (PR ) (QR )
=
4R1

8!
= 56.
5!3!

1
(5) (QR)
2
(19.5) (11) (QR )
=
4R1

Choice (A)

Choice (A)

16. The total number of ways in which one can go from A to


C = 15 C 7 .
The no. of ways in which one can go from A to C
via B = 9 C 4 x 6 C 3 .
Therefore the number of ways in which one can go from
A to C but not via B = 15 C 7 .- 9 C 4 x 6 C 3 .
= 6435 2520 = 3915
Choice (A)
17. Case
If 2 is a digit of the number, it must be of the form 24 .
Here the units digit can be filled with any of the five odd
digits i.e. by 1, 3, 5, 7, 9.
5 such numbers can be formed
Case
If 2 is not a digit of the number, then it can be any threedigit odd number not containing 2.
8 9 5 i.e. 360 such numbers can be formed.
A total of 365 such numbers can be formed.
Choice (B)

19.5
5
Q

2.

11
R

R = 21.45

Choice (4)

Let AB = 7, BC = 16 and CA = y (where y is an integer)


As ABC is an obtuse triangle, the square of its longest
side must be greater than the sum of the squares on
the other two sides.
Max(7, 16, y) = 16 or y.
If Max = 16, 162 > 72 + y2.
2
y < 207
i.e., y 14.
By triangle inequality y + 7 > 16 i.e., y > 9.
y = 10, 11, 12, 13 or 14.
If Max = y, y2 > 162 + 72.
2
y > 305 ie., y 18.
By triangle inequality, 16 + 7 > y i.e., y < 23.
y = 18, 19, 20, 21 or 22.
Total number of possibilities of y is 10. Choice (2)

3.
P

M Y

18. The total number of ways in which the parcels for Bihar
can be addressed

1
1
1

+
)
2!
3!
4!
1
1
1

+
)
= 24 (
2
6
20
12 4 + 1
)=9
= 24 (
24

15. We can have four 4 s and one 5 or one 4 and four 5 s.


So, the number of 5-digit numbers that can be formed =

5!
5!
+
4!1!
4!1!

1
(PS) (QR)
2
Let the circumradius of PQR be R1 for a triangle with
abc
sides a, b, c. Then area of the triangle =
4R1

Area of triangle PQR =

= 4! (

30
30

Let QN = 1.
NY =

3 (Similarly MX = 3 ) and PQ = 2

MY = PQ NY = 2

3 (Similarly NX = 2 3 )

The total number of ways in which the parcels for Delhi


can be addressed

Ar (XQR) + Ar (YPS) = 2(2 3 )

1
1
1
1
= 5! (

)
2!
3!
4!
5

= 4 2 3 Ar(PQRS) = 4.

21 42 + 210 147
2
2
cm
10

4 ( 4 2 3)
Ar of PQ X RSY
2 3
=
=
Re maining area
2(2 3)
2(2 3)

3
2 3

3 (2 + 3 ) = 2 3 + 3

6.

441
cm2
5

Choice (5)

The two extreme cases are shown below.

Alternative Method:
Ar of the double trapezium
Ar of the two isosceles

=
=

4.

X
Y

Ar XYPQ
Ar PMY + Ar QNX

XMN = 0

3
Ar QNYT
NY
=2 3 +3
=
=
Ar TYMP
YM
2 3
Choice (2)

A
Y

XMN = 60
0 60

C1 120 C2

Choice (3)

7.

Let C1 and C2 be the centers of the circles and A and B


be the intersection points common radius for both
circles
C1 C2 A is equilateral.
Similarly C1 C2 B is equilateral.
AC1C2 = BC1C2 = 60
AC1B = 120
Required area = 2(area of the segment AC2B)
Area of segment AC2B = Area of the sector C1AC2
B Area of triangle C1C2A
120
1
(2)2
(C1A) (CB) Sin ACB
=
2
360
=

4
1

(2)(2) sin120 =
2
3

x y y x
B
A F
C G
E

Let A and B be the centres of the circles. Let D and E


be the points of intersection of the two circles. Let C be
the intersection of AB and CD.
Let AF = x, FC = CG = y and GB = x.
Radius of each circle = x + 2y.
x + 2y
2
x + 2y
1
i.e.
.
=
=
x + 2y + x
x+y
3
3

3 sq.cm

cosCBD =

Required area =
2 3 sq.cm.
3

.
6
Common region area = 2 (Area of sector BED Area of
2

1
2
2
6

BED) = 2
.(BD) (BD) sin EBD
2

CBD =

Choice (3)
5.

CB
x+y
3
=
=
.
BD x + 2y
2

Triangles PUT and


P
PVR are similar.
PU PV

=
S
U
T
UT VR
21 7
=
=
6
2
v
Let UT = x cm
R
Q
7
PU =
x cm
2
7x
) cm
UV = (21
2
Total surface area of the cylinder
7x
= (2x) (x + 21
) cm2= x(42 5x) cm2
2

=
(5x)(42 5x) cm2
5
As the sum of the two factors 5x and 42 5x is
constant, the product is maximum when 5x = 42 5x or
21
x=
5.
The corresponding area is

]
. [ EBD =
= 2 BD 2
6

3
4

Required ratio =

(2) BD2
6

BD 2

3
4

2 3 3
.
6

Choice (D)
8.

Number of equilateral triangles which can be formed of


side a cm using equilateral triangles of side b cm
2

a
=
b
Total number of triangles used
2

16
18
30
= + ......
= 82 + 92 + 152
2
2
2

= 12 + 22 + .. 152 (12 + 22 + .. 72)


=

(15)(16)(31) (7)(8)(15) = 1100.


6

PQ = (PQ 9) + (PQ 32)


Considering PQ = x, we get
x2 82x + 1105 = 0
(x 17) (x 65) = 0
x = 65 or 17
As x > 32, x = 65.

Choice (B)

9.

8 3
=16.
sin 60
side of equilateral triangle = 32 cm.

BD =

D
Choice (B)

B
Similarly IB = r 3 .
GI = DE = sum of the
radii of the circles whose centres are D and E = 2r.
AB = r 3 + 2r + r 3 = 2(1 +

AB

= 2 1+ 3
r

Let B and C be the centres of the circles.


Let A and D be the points of intersection.
AB = AC = 8 cm.
180 A
B = C =
= 45
2
ABD = 90.
Area of the shaded region = 2(Area of the sector BAD
1

90
Area of BAD) = 2
.(BA )2 .(BA )(BD )
2

360

2 2

= 1(BA )2 =
(8) = 32( 2) sq cm.
2
2

Choice (D)

3 )r.
15.

Choice (C)

T
U9

C
D
Area of region P = 2(Area of sector EFB Area of
EFB)
1

90
=2
.(EB )2 .EF.EB
2

360
2 2

= 1(EB )2 =
(8)
2
2

Therefore the area of field accessible to the cows


(8 )2
= 2
Area of region P
2

(PQ 32)

(8 )2
Required area = AB2 2
2

2
= 16 (64 ( 2) 32)
= 256 64 + 32 64
= (192 32) sq m.

Q
32

B
P

12. Let the sides of the triangle be a, b and c. Let its area
be A.
X = abc
A
abc
a + b + c abc
=
Y =
.
.
2
4
4 A a + b + c

Y 1
Choice (A)

=
X 4

11. Number of triangles which can be drawn by using the


N points as vertices = NC3.
Number of triangles which can be drawn by using all but
one of the N points as vertices = N1C3.
N
C3 N1C3 = 210.
N(N 1)(N 2) (N 1)(N 2)(N 3 )
= 210

6
6
(N 1) (N 2) = 420
N2 3N 418 = 0
(N 22) (N + 19) = 0
As N > 0, so N 19.
N = 22.
Choice (B)

13.

ABC is equilateral such that radius = 8 3 cm

10. Let the radius of each


circle be r.
AB = AG + GI + IB.
A = 60
A
= 30.
GAD =
2
GD
AG =
=r 3 .
tan GAD

Choice (B)

[ PQ = SU]

14.

8 3
8 3
60

2 2

(8 )

Choice (C)

V
P

16.

(PQ 9)
A

S
W
PQRS is a square.
S = 90 and PQ = QR
= RS = PS
SU2 = UW2 + SW2

30
Q

R
The figure above is not to scale.

In PQA and QAR, PQ = QR.


PA = AR ( QA is the median to PR)
QA is a common side
PAQ QAR.
PQA = AQB i.e., AQB = 30.
ABQ = AQB = 30 ( AQ = AB).

Figure 1

D
Choice (B)

17. The sides given are 68 and 32.


68 = 4 17; 32 = 4 8
As 8, 15, 17 from an right angled triangle, the third side
= 4 15 = 60
Choice (D)
18. Let F and G be the midpoints on the diameters of the
cylinder.
Required volume = Volume of the cone ABC
A
1
(FC)2(AF)
=
3
AF FC
F
.
=
B
C
AG GE
AF 15
.
=
54 18
AF = 45.

A
Figure 3

Figure 2

E
C
O
D

Required volume =

In figure 2, AB = AD = 2r
BD = 2(BO) = 2r 2

Choice (D)

19. Let the sides of the cubes be a cm and b cm respectively.


4a2 + 4b2 = 468 and a3 + b3 = 945
a2 + b2 = 117 and a3 + b3 = 945 (1)
Let a be b.
Then a3 b3.
(1) 2a3 945 a3 472.5. Also a3 < 945. (1)
a is an integer. a3 is a perfect cube
(1) a3 = 512 or 729.
a3 = 512 b3 = 433. a3 = 729 b3 = 216.
b is an integer. b3 is a perfect cube.
b3 = 216 and a3 = 729 i.e., b = 6 and a = 9.
3375(2)
2
(9 + 6)3 =

Volume of the hemisphere =


3
3
= 2250 cubic cm.
Choice (B)

20.

(2r+r 2 )

G
D

3
2
(15) (45) = 3375 cubic cm.

12

BO = r 2
Now, in EOB, EO OB
EB = 2r (The distance between the centers of two
spheres touching each other)
EO =
=

(EB)2 (OB)2

(2r )2 (r 2 )2 = r 2

Figure 3 shows that the height of the structure must be


greater than or equal 2r + r 2

= (2+ 2 ) 3 2 =

6 2 +6

Choice (A)

22. We are free to choose the points as close to each other


as possible. There is no lower limit to the distance
between any two points. (We can reject choices B
and C) we have to try to spread the points out as far as
possible, i.e, choose the 3 vertices and the centroid.
This particular choice of 4 points satisfies the conditions
in choice A (while any other choice of points would
satisfy both A and D).
Choice (A)

40

32

23. Let the length and breadth of the rectangle MNOP be l


and b respectively where l and b are integers.

20
20
B

24

Length of MO = Length of PN = l + b
Joining the midpoints of a rectangle we will give a
rhombus whose area is half the area of the rectangle.
2

E C
4

322 = 1024, 242 = 576 and 402 = 1600.


322 + 242 = 402.
The triangle is right angled at B.
AC
= 20.
Circumradius of ABC =
2
AD : EC = 12 : 4 = 3 : 1.

21. Let PQRS be the base of the cuboid. PQ = QR = 6r =

18 2
The bottom layer is shown in the Figure 1 and the
dotted lines represent the second layer. The relative
position of the centers of 4 balls in one layer and the
center of the fifth ball which fits into the depression that
is formed is shown in Figure2. Figure 3 gives the front
view from which we can get the height of the cuboid.

N
B

D
Choice (A)

O
1
Area of the rhombus ABCD is
Area of PQRS
6
1
=
(54 ) = 9
6
P

The area of the rectangle MNOP has to be 18.


18 can be written as the product of two integers in 3
ways
18(1), 9 (2), 6(3).

From these 3 pairs of factors, only 6(3) is possible as


the other options imply that MN > 9.
l = 6, and b = 3. Therefore the measure of the

Therefore 8 x are right or obtuse. (All the measures


are taken in degrees)
The sum of the x acute angles < 90x
The sum of the 8 x non acute angles < 180 (8 x)
The sum of all the 8 angles = 6(180) < 90(x + 16 2x) =
90(16 x)
6(2) < 16 x x < 4
x is at the most 3.
Choice (C)

l2 + b 2 = 6 2 + 3 2 = 3 5
NP 3
AB = BC = CD = AD =
=
5 .Choice (B)
2
2

diagonal of MNOP is

24. The different possibilities for the radii of the two circles
are as follows.

27.

B
C

F
O

E
A

Since AB is a chord of the outer circle, a perpendicular


dropped from the centre will bisect it.
AC = CB and OC AB
Again in triangle ACO
(AC)2 + (OC)2 = (OA)2
or a2-b2 = 202 24 52 where OA = a, OC = b
4 2
(ab) (a+b) = 2 5 (There are 15 factors or 8 pair of
factors)
= 2(200) = 4(100) = 8(50) = 10(40)
The factors in each pair have to be both distinct and
both even.
2a = 202, 104, 58 or 50. The diameter of the bigger
circle can be 50, 58 or 104, but it cannot be 96.
Choice (C)
25.

B1 to B6 are at A to F respectively. Their speeds and the


number of rounds completed in the time taken by B6 to
complete 3 rounds are tabulated below.
B1
1
0.5

Speed
No of rounds

B2
2
1

B3
3
1.5

B4
4
2

B5
5
2.5

B6
6
3

B1 would be at D
B2 would be at B
B3 would be at F
B4 would be at D
B5 would be at B when
B6 completes 3 rounds and is at F.
There would be no boy at A, C or E. Choice (C)
28.

P
C
A Q

F
G
A

PQ = 54 = 2(3 ), QR = 240 = 2 (3)(5)


The GCD = 2(3)
PQ = 6(9), QR = 6(40)
PR = 6(41) (92+402 = 412)
i.e, PR = 246
QA + DE + HI =

3
1
(QR) =
(QR) and BC + FG + JR =
6
2

3
1
(PR) =
(PR)
6
2
[since BB1 + CC1 + FF1 + GG1 + JJ1 + RR1 = JI + GH +

XB + YD

7
FE + CD + BA + PQ
(PQ)].
2

AP + CR

The total distance travelled

= 189 + 120 + 123 m = 432 m

2 XB
2 AP

XB

26. All the angles of convex octagon are acute, right of


obtuse (but not reflex). Let there be x acute angles.

Choice (C)

z
A

Y
D
Choice (D)

=1

AP

29.

1
1
1
= [7PQ] + [QR] + [PR]
2
2
2
1
240 246
=
m
[7 54] +
+
2
2
2

As AP = AQ = CR = CS, each acute angle in APQ and


CRS is 45. Similarly each acute angle in DPS and
BQR is 45, i.e, PQRS is a rectangle.
Also as AP = CR and AP || CR,
APCR (not shown) is a parallelogram and the diagonals
bisect each other i.e., the midpoint of PR is the centre
of the square
If U is such that BU = AP PAU UBR and PU = UR,
i.e., U is equidistant from P and R
(no other point on AB can be equidistant from P and R)
The perpendicular bisector of PR (say L) intersects AB
at U.
U lies on p and AB
X lies on p and AB X = U
Also DY = BX and

J
Q

X
B

30
G

In AGB, G =

YOC ~ ABC
YO : OC : CY
= AB : BC : CA
Let OY = 1 therefore BC = 2
(The crease is equal to the length)
Let OC = d therefore AC = 2d
Let AB = x

360
= 120
3

AG = BG A = B = 30
Alternate solutions
Let GX =1 AX = 3 , CA = 2, GY = GZ = 2+ 3
Let the radius of the circle with centre at D be r1
Consider DAX, DX =GY YD GX = (2 +

Therefore 1 : d:

3 ) r1

d
d2 + 1
=
d2 = d2 + 1
2
2d
1 d
2
Also
= x=
x 2
d

1= 1+ 3 r1

Therefore

3 and AD = Sum of radii =

AX =

d2 + 1 = x : 2 : 2d

3 +r1

AD = ( 3 +r1) = ( 3 ) + (1+ 3 r1)2


2

3 + 2 3 r1 + r1 2

= 3 + 4 + 2 3 + r12 2r1 2

r1 (2 + 4 3 ) = 4 + 2 3 r1 =

3 r1

1+ 5
2

2+ 3
1+ 2 3

2 2

1+ 5

5 1

2 2
2

= 2 5 2

Therefore, the ratio of the breadth to the length is

(2 + 3 )(2 3 1)
(4 + 3 3 )
=
=
11
11

2 5 2
=
2

5 1
2

Choice (D)

Also the radius of the smallest circle centred at G


33.

= AG AX = 2 3

4+3 3
1
Required ratio =
11
2 3

( 4 + 3 3 )(2 + 3 ) 17 + 10 3
=
=
11
11

Now (PT) (TQ) = (RT) (TS)


(D) (18) = (RT) (2 RT) RT = 6. TS = 12.
Let O be the centre of the circle. The perpendiculars
dropped from O to PQ and RS meet the respective lines
at M and N.
Now PM = MQ = (PT + TQ ) / 2 = 11

From statement , the ratio of the sides of the triangle

1 1 1
: :
= 9 : 4 : 3. The greatest side is more than
4 9 12

the sum of the other two sides only in is true, is


false.
Choice (A)

H
20

Similarly RN = NS =

x
P

(3)2 + (11)2 =

OQ =

40

34.

602 EC2 = 402 BE2 = PE2


EC2 BE2 = 602 402
Again 202 AH2 = x2 HD2
x2 = 202 + HD2 AH2
Now, HD = EC and AH = BE
x2 = 202 + HD2 AH2
x2 = 202 + EC2 BE2
x2 = 202 + 602 402 = 2400

Choice (C)

P
30

75

B
75

Choice (B)

130

15

x = 20 6
32.

RT + TS
=9
2`

NT = TS NS = 12 9 = 3
Again NT = MO
In OMQ , (OQ)2 = (OM)2 +(MQ)2

60

1 1 1
the ratio of
: :
= 15 : 10 : 6
6 9 15

31.

Choice (B)

30. From statement , the three sides of the triangle are in

is

T
N

As APD : DPC : CDB = 2 : 5 : 5


APD = 30, DPC = 75, CPB = 75
PCD = 75, and PD = DC
In DAP, A = 90, D = 30
PD = 2AD hence DC = 2 AD
Or AB : BC = 2

Choice (C)

35. Let the base of the cuboid have a side 'b' and let the
height of the cuboid be h

The length of the longest diagonal = b + b + h


2

The square of the length of the longest diagonal =


2b2 + h2
It is given that 2b2 + h2 = 2502.

If b is even and h is odd then 2b2 + h2 will be odd.


If both b and h are odd, then 2b2 + h2 will be odd
So we can conclude that h is not odd, i.e. h is even,
with the different values of h that we get.
H2
4
16
36
64
100
144
196
256
324
400
484
576
676
784
900
1024
1156
1296
1444
1600
1764
1936
2116
2304
2500

H
2
4
6
8
10
12
14
16
18
20
22
24
26
28
30
32
34
36
38
40
42
44
46
48
50

(2502 h2)/2
1249
1243
1233
1219
1201
1179
1153
1123
1089 (332)
1051
1009
963
913
859
801
739
673
603
529 (232)
451
369
283
193
99
1 (12)

Area of pentagon APBCD = 2x + y


3 (2x + y) = 7y

1
( AP) (PD) sin APD
2
2
=
1
3
(DP) (PC) sin DPC
2
sinAPC = sinDPC.

Choice (C)

a+b

12

13

15

17

a+b

30

40

50

10

60

70

20

48

52

60

68

As r = 8, a = 20
Alternate solution: Let the sides of the triangle be a, b
and c, respectively.

D
8 M 9
P
O

Choice (A)

The s for the given triangle is 60


This makes us think of the following

36.

AP
2
= .
PC
3

38. Let the lengths of the 3 sides, the inradius and the
semiperimeter be a, b, c where a b < c, r and s
respectively. It is easy to verify that r = s c = a + b s
Method
Two sets of values of the 6 quantities a, b, c, r, s and
a + b are given below.

Thus three such cuboids are possible.

x 2
=
y 3

6x = 4y

a+b+c
=60 where c=
2
1
triangle =
ab = rs
2
1
Now,
ab = 8(60).
2

s=

81.25

B
Let O be the centre of the circle. Let M be the midpoint
of AC and B be that of BD.
OM AC and ON BD. MC = 8.5 and
PM = 0.5.
OM2 = OC2 MC2 = 81.25 72.25
OM = 3
Let BN = x, DN = x
PB = x + 3, PD = x 3
The chords AC and BD intersect at P
(x + 3) (x 3) = 8 (9) x = 9
PB = x + 3 = 12 and PD = x 3 = 6
Choice (D)

As r = 8 and s = 20, c = 52. a + b+ a + b =120.


ab = 960 -----(1) a + b = 68 ------(2)
Solving (1) and (2), we get a = 20 and b = 48.
Therefore the smaller side measures 20.
Choice (B)
2

39.

7.5

37.

a2 + b2 and area of the

B
4
P

Z
y

D
Let the area of BPC be z
Let the area of APD = x
and the area of PCD = y
Area of ADC = Area of BDC
X+y=z+y
z=x

Lower A. Show AB = BP Also PD = DC


APC = 90 and AQ = CQ
APB and CPD are complements
Let BP = x
PD = 11.5 x
tan APB =

7.5
4
and tan CPD =
x
11.5 x

As the angles are complements

x + 10 must be divisible by 32.


(2) x + 10 = 32 or 64 or
96 i.e., x = 22 or 54 or 86.
If x = 22, (3) y = 93
5(x + 10 )
(3) y = 4x +
32
y > 4x.
(2) 99 > y > 4x i.e.,
x < 24.75.
x = 22 is the only possibility.
y = 93
y.x = 93.22. The amount left with him after he bought
the biscuit is 91.72.
Choice (3)

4
7 .5
.
=1
x 11.5 x
30 = 11.5x x2
2
2x 23x + 60 = 0
(x 4) (2x 15) = 0
x = 4 or 7.5
If x = 7.5, BP = 7.5, PD = 4 and ABP PDC i.e AP =
CP i.e P would also be the point that is equidistant from
B and D i.e P = Q and PQ = 0
If x = 4, BP = 4 and BQ = 7.5, i.e PQ = 3.5
Either PQ = 0 or 3.5
Choice (D)
40. PQ = QR [since it is an is isosceles triangle] and PR =

PQ 2 = QR 2

3.

It is given that PR = (QD) 2


Taking Q as the centre we can draw a circle with
radius equal to QR as QD = QR, the point D lies on the
major arc.
PDR =

9(b a)
9
18
27
36
45
54
63
72

1
PQR [Angle at the center is two times the
2

angle at the circumference]


PDR =

90
= 45
2

Let the first number seen by Nagu be ab. The second


number must be ba, or the reverse of the first number.
The difference is 9 (b a).
The third number was ba + 9(b a). The sum of the
digits of this number is a + b. We can tabulate some
values of these 3 numbers. We consider successive
values of 9 (b a) and try to write down one set of
values for
ab, ba, ba + 9 (b a)

Choice (C)

ab
12
24
36
37
16
17
18
19

ba
21
42
63
73
61
71
81
91

ba + 9 (b a)
30
60
90
109
106
125
144
163

Special Equations

We see that all possible values of b a (1 to 8) are


realisable.
Choice (D)

Solutions for questions 1 to 3:


1.

2.

Let the number of 1-femto, 2-femto and 20-femto notes


with him be x, y and z respectively.
x
+ 2y + 20z = 47
(1)
(3)
(2)
(3)
(2)
(5)
(1)
(7)
(0)
------------------------------------(1)
(13)
(1)
(3)
(12)

(27)
(0)
------------------------------------(1)
(23)
(0)

(47)
(0)
-------------------------------------We can see that there are 4 + 14 + 24 or 42 ways to
settle the bill.
Choice (5)
Let the amount on the cheque be Rs.x.y.
Amount received by Mohan = Rs.y.x
Amount left with Mohan after he bought the biscuit
= Rs.(y.x 1.50)
4x.y = y.x 1.50
(1)
In Rs.x.y, y represents the number of paise.
0 < y 99
In Rs.y.x, x represents the number of paise
0 < x 99
0 < y, x 99
(2)
(1) 4(100x + y) = 100y + x 150
399x = 96y 150
133x = 32y 50
128x + 5(x + 10) = 32
(3)
i.e., 5(x + 10) = 32(y 4x)

Equations, Ratio, Proportion, Variation


Solutions for questions 1 to 6:
1.

2.

1
m
+ 1
After A gives to B, he has = m m 1 =
2
2

After giving to C, he has


1m
m 3
m

+ 1 + 1 1 =
+
2
4 2

2 2
After giving to D, he has
1 m 3
m 3
m 7
m 7
+ 1 =

+
+
+ =4
2
4
2
8
4
4 2
8 4

m = 18 Choice (5) follows.


Choice (5)

Given a

1
b

b c2

b = k2c2
ab = k1
b = 3, a = 4, k1 = 12
Given b = 3 when c = 6
3 = k2 : 36 k2 = 1/12
k
ab = k1 a = 1
b
12
4
=
a=
9
27
b
27
2
b = k2 c c2 =
=
k 2 1 / 12
c2 = 27 12
c = 9 2 = 18
9a + 5c = 9

4
+ 5 18 = 94
9

Choice (A)

3.

Given 3a + 5b + 7c = 1.25 k
2a + b + 3c = 0.75 k

(1) 2 (2) 3 gives

(1)
(2)

Time and Distance


Solution for question 1:
1.

6a + 10b + 14 c = 2.5 k
6a + 3b + 9c = 2.25 k

7b + 5c =

4.

5.

6.

and QP = 420 3
Time of arrival of the train
at Q

7b + 5c = 0.25 k
1
k
4
7b + 5c is 25% of k

As PQ = 840, RP = 420

Say Abilash has answered x questions correctly and y


questions wrongly.
x + y > 85
(1)
4x 2y = 70
2x y = 35
2x = y + 35
x + y > 85
2x + 2y > 170
y + 35 + 2y > 170
3y > 135; y > 45
when y is an even number then the number of marks
obtained by Abhilash cannot be 70.
y is always odd which is greater then 45 If y = 47
4x 2y = 70
4x = 70 + 94
164
x=
; x = 41
4
The minimum number of questions that Abhilash
answered correctly is 41.
Choice (D)
Let Arun, Varun, Kiran have initially Rs.a, Rs.b,
Rs.c respectively
Given a + b + c = 8000
Let after spending their amount they have Rs.x, Rs.y,
Rs.z respectively
Given x : y : z = 14 : 22 : 15
x = 14 k; y = 22k; z = 15k
(x + 50) + y + 100z + 200 = 8000
x + y + z = 7650
14 k + 22k + 15 k = 7650
51 k = 7650
7650
k=
= 150
51
y = 22k
= 22 150 = 3300
The amount that Varun had initially is 3300 + 100
= Rs. 3400
Choice (B)

hours + 9 a.m. =

84 3
2 p.m.
he latest time by which
Raju has to arrive at Q
= 1:35 p.m.
The time he takes to cover
840
RQ =
210

Choice (A)

Let the bag contain x two rupee coins and y five rupee
coins
Let 2x + 5y = k
(1)
75
Again given 2x + 5(3y) = k +
k
100
7k
2x + 15y =
(2)
4
(2) (1) given
3k
10y =
4
3k
y=
40
when k is multiple of 40 then y is multiple of 3
when k = 240
y = 18
The number of five rupee coins may be 18.
Choice (C)

420 3

60

420
P

840

30

420 3

3
= 4 3 hours = 6 hr 56 min.
The latest time at which Raju can leave
R = 1:35 p.m. 6 hr 56 min = 6:39 a.m. 6:40 a.m.
Choice (2)
Solutions for questions 2 and 3:

Let the cruising speed of the plane be S kmph.


Let the time difference between P and Q be t hours.
Time taken by the plane to travel from P to Q
= 5:00 p.m. t hours 7:00 a.m.
= (10 t) hours.
Similarly time taken by plane to travel from Q to P
= (10 + t) hours.
Effective speed of a plane traveling from P to Q
= (S + 100) kmph.
Effective speed of a plane traveling from Q to P
= (S 100) kmph
4800
10 t =
S + 100
4800
10 + t =
s 100
Solving for S and t,
S = 500 and t = 2
2.

3.

Required time = 5:00 p.m. t hours


= 3:00 p.m.

Choice (2)

500 km/hr

Choice (3)

Solutions for questions 4 to 8:


4.

After B travels for 48 min, (and covers 24 km) let us say


A reaches Q and B reaches R.
P
Q
24

74

As QR = 2 (37), PR = 2 (12)
PQ = 2 372 122 = 2 (5) (7) = 70
To cover 70 km, A must have travelled for

70
hrs or
45

5
1
1
hr. = 1 hr 33
min = 93
min.
9
3
3
To cover 24 km, B has travelled for 48 min.
1

10

B started (93

9:00 a.m.
B started at 9:45:20 a.m.
5.

6.

Choice (B)

The distances that A, B, C run by


finish the race are tabulated below:
A
By the time A finishes
x
By the time B finishes
x 180 x 351

=
x
x 198
2
2
x 378x + 180 (198) = x 351x
27x = 180 (198)
x = 20 (66) = 1320

8.

Choice (D)

can be

13
6 (14 ) 28
=
=1
15
45
15

6 1
52
100% or by
100%
6
5
i.e, by 831/3% or 60%.
Thus, the value of P can be 831/3% or 60%.
Choice (D)
Quadratic Equations
Solutions for questions 1 and 2:
1.

Let the numbers (in ascending order) be L, L + 1, L + 2.


Given 3[L3 + (L + 1)2 + (L + 2)] = (3L + 3)2
3
2
2
3(L + L + 3L + 3) = 9(L + 2L + 1)
3L3 6L2 9L = 0
L =) or L2 2L 3 = 0
L(L + 1) (L 3) = 0
L = 0, 1 or 3.
As L > 0, L = 3
Choice (1)

2.

For any cubic equation in x, in which the coefficient of x3


is 1, the coefficient of x is equal to the sum of the product
of the roots taken two at a time.
q = ab + bc + ca. As a, b and c are integers,
q must be an integer.
Let us go by the choices. Choice (5) is the least among
the choices but is not an integer.
it is ruled out. Choice (4) is the second least.
If q = 1, ab + bc + ca = 1.
This is possible if a = 1, b = 0 and c = 1 and if
a = 1, b = 0 and c = 1.
The least possible value of q = 1.
Choice (4)

Choice (D)

We can divide the track into 5 units (1 unit = 30m)

6
5
or .
1
2

So Sanjay's speed can be less than Ajay's by

While Arun covers AB, the bike covers AB + BC + CB.


AB
AB + BC + CB

=
5
45
9AB =(AB + BC + CB) -------(1)
While Tarun covers CD, the bike covers CB + BC + CD.
9 CD = CB + BC + CD ------- (2)
Consider Arun and Tarun.
Each covers part of the distance at 5 km/hr and the rest
of 45 km/hr. Both take the same time to cover AD.
Each has to cover the same distance at the lower
speed (and hence the same at the higher speed) i.e.,
AB = CD.
Also BC + CB = 8 AB (from 1) or BC = 4 AB.
Alternately, (1) and (2) AB = CD.
ab : bc : cd = 1 : 4 : 1
AB = 6, BC = 24, CD = 6
The time taken (in hours) is (considering Arun)
1
6 30
1
+
= 6
+
5 45
9
5

a
, then Ajay and Sanjay will meet at
s

(a + s) distinct points on the track when travelling in


opposite directions and (a s) distinct points on the
track when travelling in the same direction.
It is given that a +s = 7
The different possibilities are 6 + 1, 5 + 2 and 4 +3.
For a - s = n, we get the corresponding possibilities as 6
1 = 5, 5 2 = 3 and 4 3 = 1
As it is given that n is a prime number, n can be 5 or 3.
Therefore the ratio of the speeds of Ajay and Sanjay

B
C
x 180 x 351
x
x 198

If the ratio of the speeds of Ajay and Sanjay in the


lowest terms is

the time A and B

Varuns bike travels at 45 km/hr. Arun and Tarun walk


at 5 km/hr. Arun covers AB on foot. Tarun covers CD on
foot.

7.

Difference = 15 units = 15 (30) m = 450 m


Choice (B)

1
1
48) min after A or 45
min after
3
3

Solutions for questions 3 and 4:

The sixth meeting is exactly like the first one (same


location, same direction for both. Hence, the pattern
repeats) The 8th meeting corresponds to the 3rd
Ratio of the speeds of P and Q = 60 : 40 = 3 : 2.
Time Interval Distance P
Start to M1
M1 to M2
M1 to M8
Start to M8

3
6
42
45

Covered Q (in
units)
2
4
28
30

4 is a root of g(x) = 0. Let the other root be y.


g(x) = c[(x 4) (x y)] where c is a constant
= c[x2 (4 + y)x + 4y].
Also 8g(3) = 3g(6)
8[c(9 (4 + y) (3) + 4y)]
= 3c[36 (4 + y)6 + 4y]
As c 0, 24 + 8y = 36 + 6y
y = 6
g(x) = c[x2 2x 24]
No information is available to find c.
p, q and r cannot be found
3.

The other root of g(x) = 0 is 6.

Choice (3)

4.

p + q + r cannot be found.

Choice (5)

11

Solutions for questions 5 and 6:

The data and calculations are tabulated below.


No.
x
10
15
20

Cost
200+ px + qx2
200 + 10p + 100q
200 + 15p + 225q
200 + 20p + 400q

Cost

9a(say) (1)
17a (2)
28a (3)

(2) (1) 5p + 125q = 8a


(3) (2) 5p + 175q = 11a
3a
a
q=
and p =
50
10
a
3a
(1) 200 + 10 + 100
= 9a a = 100
10

50
p = 10, q = 6
The cost of production of x items is 200 + 10x + 6x2
The selling price is 100x.
The profit is p = 6x2 + 96x 200.
When a < 0, the expression ax2 + bx + c has a maximum
b
value and it occurs when x =
.
2a
96
The profit is maximum when x =
= 8, and the
12
maximum profit is 6(64) + 96(8) 200 = 184.
5.

6.

The maximum profit is Rs.184.

E(y) = ay2 + by + c
Where a, b and c are constants.
E(y) is minimum when y = 3.
b

= 3
2a
i.e., b = 6a
E(3) = 1
1 = 9a 3b + c
1 = 9a + c
(1)
E(2) = 2 2 = 4a 2b + c
2 = 8a + c
(2)
Solving (1) and (2), a = 1
And c = 10.
b = 6 and hence E(y) = y2 + 6y + 10.
E(4) = 50.

Choice (A)

11. Let , are the roots of the equation x2 (k + 7) x


(3k 15) = 0
+ = k + 7; = 15 3k
( )2 = (+)2 4
= (k + 7)2 4 (15 3k)
= k2 + 14 k + 49 60 + 12 k
= k2 + 26 k 11
2
(k + 13) 11 169
2
( ) = (k 13)2 180
The minimum value of ( )2 is 180.
Choice (B)
12. The data is tabulated below:
Cost of each
cup

For the profit to be maximum, the daily production has


to be 8.
Choice (3)

Number of cups
Total Cost
of tea

90

x + 1.50

n 10

90

Choice (5)
To get rid of the fraction, we can multiply by 2.

Solutions for questions 7 to 18:


7.

10. Given = 9
|| || = 5;
Squaring on the both sides
2 + 2 2 | | = 25 (1)
2
2
= 9 + 2 = 81
2
2
+ = 81 + 2 (2)
Substitute the value of 2 +2 in (1)
81 + 2 2 || = 25
81 25 = 2 || 2
56 = 2 2 ( is negative)
56 = 4
= 14

Cost of each cup Number of cups


Total Cost
(Hypothetically)
of tea

2x

180

2x + 3

n 10

180

We see from the options that only integral values of 2x


have to be considered. 180 = 22 (32) (5). There are 3 (3)
(2) or 18 factors or 9 pairs of factors. They are
1 (180)
5 (36)
2 (90)
6 (30)
3 (60)
9 (20)
4 (45)
10 (18) and 12 (15)
Choice (3)

8.

Since when x = 1; both the equations become a + b + c = 0


1 is the common root of the equations
a+b+c=0
3
3
3
a + b + c = 3abc
Choice (B)

9.

We know that if , , are the roots equation


x3 + p1x2 + p2x + p3 = 0; then + + = p1
+ + = p2
= p3
Given , , are the roots of the equation,
x3 + 2x3 5x 6 = 0
+ + = 2;
+ + = 5
=6
Now given , , are the roots of the equation
x3 + px2 + qx + r = 0;
+ + = p = 5 = p
p=5
Choice (C)

We need to look for two numbers that differ by 3 (in the


left column) and the corresponding numbers in the right
column should differ by 10.
2x = 6 and n = 30
follows ( 2x + 3 = 9 and n 10 = 20)
The price is Rs.3 per cup.
Choice (A)
13. x4 + y4 2x2 y2 32x2 32y2 + 256
= (x2 y2)2 32(x2 y2) + 256 64y2
= (x2 y2 16)2 (8y)2
= (x2 y2 8y 16)(x2 y2 + 8y 16)
= [x2 (y2 + 8y + 16)][x2 (y2 8y + 16)]
= (x + y + 4)(x y 4)(x + y 4)(x y + 4)

Choice (C)
14. The maximum value of the expression will occur at x =

b
.
2a

12

It is given that

b
=1
2a

b = 2a
The value of the expression at x = 0 is c.
It is given that c = 1
So the expression is ax2 2ax 1
It takes a value of 5 at x = 1
a(1)2 2a(1) 1 = 5
a=6a=6
The expression is 6x2 + 12x 1
The value of the expression at x = 5 is 6(5)2 +
12 (5) 1 = 211
Choice (D)
15. The roots of the equation are k kr, kr2
It is given that k2 (1 + r2 + r4) = 364 = 4(91)
Since the roots are integral, k2 can be only 1 or 4
364 = 1 (364) or 4(91)
If k2 = 1, then 1+r2 + r4 = 364
2
4
r + r = 363
r2 (1 + r2) = 363
i.e. product of 2 consecutive values to give 363 which is
not possible.
k2 = 4 and 1 + r2 + r4 = 91
r2 (1+ r2) = 90 = 9(10)
2=
r 9 r = 3.
The product of the roots is c i.e. negative. Thus, the
roots could be 2, -6, 18 or 2, -6, 18
In either case c = 2 and hence product = 216.
Choice (D)
16. Let the roots be ,
+ = 6 ---- (1)
2 + 2 = 16 ---- (2)
(1) 2 + 2 + 2 = 36
2 = 20, = 10
Choice (B)
Note: The equation would be k (x2 6x + 10) = 0
17. x 21x + px 280 = 0
Let the roots be , ,
+ + = 21 and , , are in AP
= 7
Let = k, = + k
The product of the roots = ( k) ( + k) = 280
72 k2 = 40
k = 3
The roots in the increasing order of magnitude are 4, 7, 10
p = + +
= 4 (7) + 7 (10) + 10 (4)
= 28 + 70 + 40 = 138
Choice (A)
3

common difference of 4.
the sequence of common terms will be in AP whose
common difference is LCM. (5, 4) i.e., 20.
Let there be N terms common to the given sequences,
where the Nth term = 29 + (N 1) (20)
= 9 + 20N. This cannot exceed min (474, 485) or 474
i.e.9 + 20N 474
N 23.25
N must be the greatest natural number 23.25 i.e.,
23.
Choice (5)
2.

Solutions for questions 3 and 4:

When x is even, we have to multiply the previous term in the


other sequence by M to get the xth term in either g or h.
When x is odd (and 3) we have to multiply the previous
term in the other sequence by N to get the xth term in either
g or h.
We will find it convenient to tabulate the terms of g and h
in 2 columns as shown below.
g
M
MN
M2N
M2N2
M3N2
M3N3
M4N3

18. In the given equation


ax3 + bx2 + cx + d = 0
sum of the roots = b/a

sum of the roots taken 2 at a time =


product of the roots =

3.

4.

+ N)

Choice (3)

g1 + h1 = 1
3
= 0.1875
16
3 2
2 3
g5 = M N and h5 = M N g5 + h5 = M2N2(M + N)
9
7.68
= 0.03
=
>
256 256
3 3
g6 = M N and h6 = M4N2
g7 = M4N3 and h7 = M3N4
g7 + h7 = M3N3(M + N)
40.96
27
<
=
= 0.01
4096
4096
7 is the least odd value of x satisfying gx + hx < 0.02.
Choice (3)

c
a

d
d
is negative i.e.
is positive implies d and a are
a
a
Choice (B)

Progressions / Series
Solutions for questions 1 and 2:
1.

x
1
2
(M

g3 + h3 = MN(M + N) =

d
a

of the same sign.

H
N
M2
M2N
M3N
M2N3
M4N2
M3N4

g2 + h2 = M(N + M)
g4 + h4 = M2N(N+M)
g6+h6 = M3N2(N+M)
We have gx + hx = M(MN)
choice (3) follows.

as two roots are positive, the other root must be


negative.
Thus the product of the roots = a negative quantity

Suppose the prices of the two varieties are equal on


day N.
Suppose n 200.
Then 110 + 0.2n = 97 + 0.25n
n = 260 which is not possible.
n > 200
If n = 200, price of Nilgiris coffee = Rs.150.
97 + 0.25n = 150 n = 212
2006 is not a leap year. February 2006 has 28 days.
212 = 31 + 28 + 31 + 30 + 31 + 30 + 31. on July 31st,
prices of the two varieties of coffee would be equal.
Choice (3)

Let the number of common terms be N. The first AP has


a common difference of 5, while the second has a

Solutions for questions 5 to 17:


5.

90

Let the series be

x = 1 ( x + 9 ) ( x + 10 ) ( x + 11) ( x + 12)

Consider f (x) =

1
( x + 9) ( x + 10 ) ( x + 11) ( x + 12)

13

1 1
1
1 1
1

6 x + 9 x + 12
2 x + 10 x + 11

90

x =1

1 1
1 1 1
1

6 x + 9 x + 12 2 x + 10 x + 11

9.

=
1 1 1 1 1 1 1 1 1 1 1
1
1
+ + + + + ... +
6 10 13 11 14 12 15 13 16 14 17
99 102

1
1
1 1 1
1
1

+ ... +

100 101
2 11 12 12 13

1 1
1
1
1
1
1 11
1
+
+

6 10 11 12 100 101 102 2 11 101

1 1
1
1
1
1
1 1 101 11


6 10 100 11 101 12 102 2 101 11

1 9
90
90
1 90
+
+

6 100
101 11 12 . 102
2 101 11

90
6

23 + 1
23 1
2 +1
=
2 1

3 4 5
3 7 13

........
.......
1 2 3
7 13 21

1
3
=( ) (3) =
2
2
=

Choice (C)

6.

The least five digit number that when divided by


8 leaves a remainder of 4 is = 10004 and the greatest
such five digit number is = 99996
The number of five digit numbers which when divided
by 8 leave a remainder 4 is 11250
11250
[10004 + 99996] = 618750000
Their sum is =
2
Similarly the sum of five digit number which leave a
11250
[10006 + 99998]
remainder 6 when divided by 8 is
2
= 618772500
Total sum = 1237522500.
Choice (D)

7.

Let a and d be the first term and common difference of


the progression
Given sn = 2400
s2n sn = 7200
s2n = 9600
n
[2a + (n 1)d] = 2400
2

n[2a + (2n 1)d] = 9600


(1) (2) we get
n[2a + (n 1)d] 4800
=
n[2a + (2n 1)d] 9600
4a + 2nd 2d = 2a + 2nd d
2a = d
a 1

=
d 2
8.

Choice (A)

2n 1
(n)(n + 1)
2n 1
3n (n + 1)
3
1
=
=

Now
n(n + 1)
n(n + 1)
n +1 n
1
3 1
=

1.2
2 1
3
3 1
=

2. 3
3 2
5
3 1
=

3.4
4 3
27
3
1
=

14 . 15
15 14

11. The general term tn of S1 is

1
3
1 1 1
+ + ... +
1+
14
15
2 3 4

Adding, we get S1 = 2

1
1 1 1
+ + + .... +

14
2 3 4

2S2= 2

(1)
(2)

2S2 S1 = 1

3
1 4
= 1 =
15
5 5

Choice (A)

12. a, b, c are in GP
Let b = ar and c = ar2
Also a12 = b15 = cx a12 = a15 r15 = ax r2x

Choice (C)

a3 =

13.

or a =

1
r5

15

x = 20

ab

2
Again t9 = a;
After every 8 times again first term be repeated.
t31 = t7 = b
t27 = t3 b

1
r 15

12

1
5
r

Let t1 = a; t3 = b
(a + b)
a+b
ba
then t2 =
, t7 = b;
, t4 =
, t5 = a, t6
2
2
2
t8 =

3 3 + 1 43 + 1

33 1 4 3 1
3 +1 4 +1
...
3 1 4 1

22 2 + 1 3 2 3 + 1 4 2 4 + 1

2
2
2
2 + 2 + 1 3 + 3 + 1 4 + 4 + 1

1
2
1
= 15

1111
1224
1000

9 28 65
..
7 26 63

1
1
3
1
+
+

1000
1111
1224
1111

1429
5666100

Choice (C)

The 1st, 3rd, 5th terms form a GP and the rest form an
AP
2 + 4 + 8 + . + 220 = 2(220 1)(G.P)
and 3 + 6 + 9 + + 60 = 630(A.P)
The required sum is 221 2 + 630 = 221 + 628
Choice (B)

10. S =

t 31 b
=
= 1
t 27
b

1
4
r

b=

1
3
r

1
r4

,c=

1
r3

20

Choice (B)

2
1
1
1 1

+
=
2
a
a
+
d
a
+
2d
a(a + d)(a + 2d)
2d
1
1
2
1

+
x1 x 2 x 3
( x1)( x 2 )( x 3 )

14

12(13) 2

12(13)(25)

=
5
9 2
6
2

= 6075 1290 = 4785


And 3 (12) +4 (22)+ 5 (32)+ +12(102) =
(1 + 2)12+(2 + 2)22+(3 + 2)32++(10 + 2) 103 = (13 + 23
+..+103) + 2(12 + 22 + 32+..102)

1 1 2 1
1
=
+
1(2)(3)
2 1 2 3
1
1 1 2 1
=
+
2 2 3 4
2(3)( 4)

1 1
2
1
1

+
=

12(13)(14)
2 12 13 14

2(10)(11)(21)
10(11)
=
= 3025 + 770 = 3795
+
6
2

1 1
1
2
1
=

13(14)(15)
2 13 14 15
S =

1
2
1
=
2
=

1 1 2 1 1
2
1

+
+ +

2 1 2 2 14 14 15
1 1 15 + 14
1
1 1
2 14 + 15 =
+ 210
2 2

26
104
1
1
Choice (A)
2 210 = 2(2)(105 ) =
105

Now,

Solutions for questions 1 to 11:


1.

1
1
1 1 1
1 1 1
= + + + .... +
1 + 2 + 3 + ... 50
100
1
2
3

2.

G(1) + G(2) = 4G(2)


= G(2)
Similarly G(3) = 900,
G(4) = 540, G(5) = 360 and
1800
G(6) =
7
3.

= f(1) [1 + f(6)] =

f(1)

10
9

10
9

5
10
2
f(1) =
=
3
9
3

10
1
1
1
f = f + f(1) + f f(1)
9
6
6
6

1
6

10
9

10
1 2
=
1 + f
6
3
9

19d
=0
2

10
2
and f(6) =
9
3

10
2
f(1) 1 + =
9
3

f(1) 1 + f =

2a + 19d = 0 i.e. a + a + 19d = 0


i.e. 1st term + 20 term = 0.
If the sum of 2 terms is 0, it follows that one of them is ai,
where 1 i 10 and the other is aj (where 11 j 20)
Choice (C)
Neither of them can be the 21st term.
2

Choice (3)

Given f(xy) = f(x) + f(y) + f(x) + f(y)


f(6.1) = f(6) + f(1) + f(6) + f(1)

n2
19d

d + n a +

2
2

G (1) = 5400
y =1

n + 19
n2
19n

d = na +
= n a +
d+
d
2
2
2

As S n2, a +

1

9
1
= =
Choice (1)
1
81

G(1) + G(2) = G(y ) = 22G(2)

16. Let the first term and the common difference be a and d
respectively
a11 = a + 10d
a12 = a + 11d
a10+n = a + (n + 9) d
2a + [10 + (n + 9)d]
The sum S = n

(G(3))
G(3 )
G(3 )
=
=
G(1)
1 G(1)
G

3 G(3 )

Choice (B)

15. The units digit of successive terms are listed below.


3
8
8
4
4
4
2
2
2
8
8
8
6
6
6
8
8
8
etc
We see a cyclic pattern of cycle length 6, except for an
initial anomaly.
i.e. t10 t16 (as well as t16 t10) and t8 t2 are all
multiples of 10 but t7 t1 is not (it is a multiple of only 5)
Choice (C)

3 G(3 )
G(3) = G =
1 G(1)
G(1) = 1



3
G(9) = G
1

3

1
1
1 1 1
1 1 1
= + + + .... +
2 + + + ... +

100
100
1 2 3
2 4 6

1
1
1
1
=
+
+
+ ... +
51 52 53
100

Choice (C)

Functions

1
1
1
1
1
1

+... +

1
2
3
4
99
100

14. S =

29
4785 319
=
=
3795 253
23

17. 1 (3 ) + 2 (4 )+ 3 (5 ) + +10(12 ) =
(3 2)32 + (4 -2)42 + (5 2)52 ++(12 2)122
= 33 + 43 ++123 2(32 + 42 + ..+122)

1 5
2
1
f =
1 + f =
6 3
6 3
4.

Given f(x) =

Choice (A)

x
1+ x

x
x
x
1 +x
f(f(x)) = f
=
=
x
1 + 2x
1+ x
1+
1+ x

15

7f(1) = 24.5 f(1) = 3.5


Now f(1) + f(2) + f(3) + f(4) + .f(7) = (1 + 2 + 3 +

x
x
1+ x
f(f(f(x)) =
=
x
1 + 3x
1+ 2
1+ x
x
1+ nx

similarly f(fn times ) =

f(f(f(f(f(f(x))))))) =
= f(f(f(f(f(f(3))))))=
5.

+7)f(1) =

Choice(C)

7.

f(x) must be a quadratic expression, since


the
difference between the values of f(x) for any two
consecutive values of x is an expression whose degree
is one less than the degree of f(x). Since f(x+1) f(x) is
a linear expression, f(x) must be a quadratic
expression.
Choice (C)

8.

The different regions on the number line and the number


of factors that are negative in those regions are tabulated
below

x
1+ 6 x

3
3
=
1 + 6.3 19

7(8)
(3.5) = 98
2

Choice (C)

f(x + 2) = f(x) + 7
f(x + 2) f(x) = 7
put x = 1, f(3) = f(1) + 7 = 9
put x = 2, f(4) = f(2) + 7 = 12
put x = 3; f(5) = f(3) + 7 = 16
i.e f(1), f(2), f(3), f(4), f(5).are as follows
2, 5, 9, 12, 16, 19, 23, 26, 30..i.e first term, third, fifth
term are in A.P with common difference 7 and second
term, fourth term, sixth term, are A.P with common
difference 7
The 150th term of 5, 12, 19, 26is t75 = a + 74d
= 5 + 74 7 = 5 + 518 = 523
f(150) = 523
The 75th term of the series of 2, 9, 16, 23.
T38 = 2 + 37d = 2 + 37 7 = 2 + 259 = 261
f(75) = 261
f(150) : f(75) = 523 : 261
Choice (A)

Region
x < 3
3 < x < 2
2 < x < 1
1<x<3
3<x

Powers whose bases


are negative in f(x)
p, q, r, s
q, r , s
r, s
S
None

As f(3.5).f(2.5)>0, p is even
As

( ) <0, q is odd
( )

f 2.5
f 0 .5

As f(0.5).f(1.5) > 0, r is even


d + e = d e d and e have opposite signs.
As f(1.5) and f(3.5) have opposite signs is odd
p, q, r, s, are even, odd, even, and odd respectively.
Choice (D)

6.

f(7) = f (1 + 6) = f(1) + f(6) and so on.


f(7) = f(1) + f(1) + f(1) + f(1) + f(1) + f(1) +f(1) = 24.5

9.

The following data is given


(a) f(0) = 5
(b) for x > 2, f(x + 2) = f(x) + 4
(c) for 0 < x < 2, f(x + 2) = 6 f(x)
(d) for x 0, f(x + 2) = f(x) 4
As f(x) is continuous, we can assume that (C) holds not only for 0 < x < 2, but also at x = 0 and at x = 2
Some values of x and the corresponding values of f(x) (or y) are tabulated below. The way y has been computed is shown
in the third column with the heading reason.
x
y Reason
0
5
a
2 9
d
2
1
c
4
5
We plot these values on a graphsheet
Y-axis

1
2

X-axis
4

16

2x 1
1
2x 1 7 x + 1
7x 1
=
f (x) =
14 x 7
14 x 7 + 14 x 2
+2
7x 1
9 x
=x
=
9

We can see that the equation of this graph is


y = f(x) = 2x 4 +1

Alternate method:
F(x+2) + f(x) = 6 for 0 < x < 2.
f(3) + f(1) = 6 (1).
Also f(0) = 5 ..(2)
(2) is satisfies only by (A), (B), and (C).
Among these, (1) is satisfied only by (A). Choice (A)
follows.
Choice (A)

fn(x) is a cyclic sequence of expressions, which is


shown below.
f1(x)
f4(x) = f1(x)
f7(x) = f1(x)
f5(x) = f2(x)
f8(x) = f2(x)
f2(x)
3
6
3
f (x)
f (x) = f (x)

Note: Students who are not familiar with continuous


functions may follow the alternative approach.

f8(x) = f2(x) =

10. f(x + 1) = f(x) + x


The values of x, the first order differences D1 [f(x + 1)
f(x)] and the second order differences D2 are tabulated
below.

a+1

a+2

a+3

a+4

f(x) b(say) b+a b + 2a + 1 b + 2a + 3 b + 4a + 6


D1

a+1

a+2

a+3

D2

For a linear expression in x, first order differences


would be a constant sequence
For a quadratic expression in x, then second order
differences would be a constant sequence
As the order differences of the given sequence, form a
constant sequence, we expect that f(x) is a quadratic
expression.
Let us verify with a simple quadratic, say f(x) = x2. The
values of x, f(x), and the differences of the first 2 orders
are tabulated below
X
2

D1
D2

16

25

36

11

Solutions for questions 1 to 5:


1.

Sum of the ages of the members twelve years ago


= 250 years.
Four years later, the sum of their ages = 290 years.
That year, a member aged 50 years died and a child
was born.
Sum of the ages of the members after the childs birth
= 240 years.
Four years later, a member aged 50 years died and
another child was born.
Sum of the ages of the members after this birth = 230 years.
Sum of the present ages of the members = 270 years.
Present average age of the members = 27 years.
Choice (2)

2.

The number, the average age and the combined age of


the two groups are tabulated below.
Group
Number
Av.Age
Combined Age
A
m
x+6
mx + 6m
B
m+3
x
mx + 3x
The difference of the combined ages is 3 (x 2m)
(The total age of B is more and it will continue to
exceed that of A). After 4 years, it would be 12 more
than this (as B has 3 more children). If 12 more than a
number is double the number, the number is 12, i.e.,
x 2m = 4. The possible values for m, (the number of
children in A), 2m + 3 (the total number of children) and
x (the average age of B) are tabulated below:

M
1
2
3

11. f =

4.

Choice (D)

Averages

Therefore, we expect f(x) to be a quadratic expression.


Choice (A)

x 1
7x + 2
x 1
1
x 1 7x 2
2
7
x+2
=
f (x) =
7x 7
7 x 7 + 14 x + 4
+2
7x + 2
6 x 3 2x 1
=
=
21x 3
7x 1

2x 1
7x 1

3.

2m + 3
5
7
9

x
6
8
10 Choice (B)

The negative deviation is 10 M + 5 (M + 2)


The positive deviation is 10 (7)
10 M + 5 (M + 2) = 7 (10) M = 4
The total number of students is 4 + 6 + 6 + 7 = 23.
Choice (D)

Initially P contains 120 L of milk and Q contains 120 L of water


Operations

P
water

total

120

120

120

120

96
96
72
81.6

0
24
18
38.4

90
120
90
120

30
24
48
38.4

120
96
102
81.6

150
120
150
120

milk
Operation 1 After transferring 30 L from P to Q
Initial contents
After transferring 30 L from P to Q
Operation After transferring 30 L from Q to P
Operation 2 After transferring 30 L from P to Q
After transferring 30 L from Q to P

Therefore, at the end of operation 2, the ratio of milk and water in vessel P is

milk

Q
water

total

M 81.6 17
=
=
W 384
8

17

Alternate solution:
The actual quantities are not important. Only the ratios
are. In each step, 1/4 of the contents of P are
transferred to Q and transferred back to P. (Q has only
4/5 of 1/4 or 1/5)
After the first operation, P is left with 4/5 of its initial
milk. Let the quantity of milk in P, initially, after one
operation and after two operations be M0, M1 M2 and the
corresponding quantities in Q be N0, N1, N2. (N0 = 0)
If M0 = 20, then M1 = 16, N1 = 4
If M0 = 20(3) then M1 = 16(5) , N1 = 4(5)
In the second operation, the quantity of milk will again
reduce the 4/5m of the starting value. But there is one
more factor to be considered. Q has some initial milk. P
gets 1/5 of that as well.
M2 = (4/5) M1 + (1/5) N1
= 4/5 (16) (5) + 1/5 (4) (5) = 64 + 4 = 68
M0 = 100, M2 = 68
The quantity of water after operation 2 is 32. (The
total remains constant)

68 17
The required ratio =
=
32 8
5.

S
4
4
4

Data Sufficiency
Solutions for questions 1 and 2:
1.

From statement A, there were 169 players. Therefore, a


player gets a bye in the first round. This may or may not
be the champion. Statement A is not enough.
From statement B, we know that the champion got only
1 bye. But we do not know the total number of players.

Choice (D)

By combining the statements, we get the following


table.
Round
1
No. of
84
Matches
Player getting
1
bye

(5 + 10 + 15 + ... + 95 )
(750)
100
19 100
19
=
750 =
(750) ml

2 100
2

42

21

11

As the champion got only 1 bye, he must have played


8 1 or 7 matchers.
Choice (4)

19
(750)
2

= 2 : 19

2.

Choice (B)
Quant Based Reasoning

Solutions for questions 1 and 2:

3
4

0
1

1
0

2
1

Round

1.

We consider the options.


(1) At least 2 horses finished before P, i.e. p = 3, 4 or
5 possible.
(2) There were 3 horses between Q and P. Possible
(Q, P) could be (1, 5) or (5, 1)
(3) T finished last is possible
(4) S came second is possible
(5) There were 3 horses between T and R. Not
possible i.e. {T, R} cannot be {1, 5} or {5, 1}
Choice (5)

2.

Using the above conclusions and the statement that S


came fourth we get the following cases.

Paired player 252 126 62

32

16

Player getting
bye

Positions
R
T

In round 1, there were N players, where 129 < M < 256.


From statement B, 1 player received a bye from round 3
to round 4.
This is not enough to determine M. (There could have
been byes in earlier rounds.)
From statement B, only one player received a bye. But
this could have been in round 1 (say of M = 131) or
round 2(say of M = 130) etc.,
By combining the statements, we get the following
table.

The ratio of the amount bet on Q, R, T is 1 : 2 : 3. There was


no gain and no loss. We have the following possibilities.
The first table shows the bets amounts on the 3 horses and
the different ways in which Mohan could have got back the
total amount he bet.
The second table shows the corresponding positions of the
horses. indicates 4th position or 5th position.

Bet
Return
factor

R
2

Where indicates 4 or 5 (As s = 4, means 5)


We see that (1) is not possible as {Q, T} = {4, 5}
S cannot be 4.
Statement (5) cannot be true.
Choice (5)

The total quantity of alcohol drunk = 750 ml.


The total quantity of water taken = Total consumed
quantity Total quantity of alcohol quantity
= [5% (750) + 10% (750) +... + 95% (750) + 100%
(750)] 750

Therefore the required ratio = 750 :

2
1

(1)
(2)
(3)

We can fill up this table (from right to left) and

[(

) ]

determine that N = 252 26 1 22

Choice (4)

Solutions for questions 3 to 6:


3.

The data is tabulated below.


Section
B BA
A
Av. Height
b
140
a
No.
40 80
40
If Mohan goes to B and Sohan to A, the average height
of the two sections would interchange. This means the
average height of the 39 other students in B (other than
Sohan) is equal to the average height of the 39 other
student in A (other than Mohan).
From (A), ba = 2. b = 141 and a = 139. (As the
number of students in the two sections is equal, the
deviations of the average height from the average of the
entire class are equal). In B, if we replace Sohan with
Mohan, the average height becomes 141, i.e. Mohan is
80 cm taller than Sohan. But we cannot get the height of

18

either.

be 3. So it is not possible, thus he failed.


Hence statement B alone is sufficient.
Choice (2)

From (B), we can tabulate the data as shown below.


Section

B-Sohan

Percentage, Profit & Loss

A-Sohan Mohan, Sohan

Av. Height

140

140

140

No.

39

39

Solutions for questions 1 and 2:

As the average height of B (without Sohan) is equal to


the average height of (A and Sohan), each is equal to
140. As the average height of the 39 students of A
(except for Mohan) is equal to the average height of the
39 students of B (except for Sohan) this average is also
140.
If we combine the two statements, we can conclude
that Mohans height is (140 + 40)cm and Sohans height
is (140 40) cm.
Choice (3)
4.

5.

6.

Using statement A, volume of the material used to


36000kg
= 9 m 3
make the tank =
4
kg / cc
1000
4
Volume of water the tank can hold =
(63) 9
3
= 23 79m3 = 279 kilolitres.
the tank will meet the companys requirements.
A is sufficient.
Using statement B, if the tanks inner radius is 4.5 m,
4
volume of water it can hold =
(4.5)3
3
= 121.5 m3 = 121.5 kilolitres
If its inner radius is close to 6 m, it can hold
approximately 288 kilolitres of water.
we cannot be sure of whether the tank will meet the
company requirements. B is not sufficient.
Choice (1)
Using statement A,
PQ + QR + PR = P2 + R(2P)
We do not know the values of P, Q and R.
We cannot find the maximum value.
A is not sufficient.
Using statement B,
(P + Q + R)2
= P2 + Q2 + R2 + 2PQ + 2QR + 2PR (1)
(P Q)2 + (Q R)2 + (P R)2 0
P2 + Q2 + R2 (PQ + QR + PR)
(1) (P + Q + R)2 3(PQ + QR + PR)
maximum value of 3(PQ + QR + PR)
= (P + Q + R)2 = 842.
maximum value of PQ + QR + PR can be found.
B is sufficient.
Choice (2)
Using statement A, we do not know anything about the
locations of A, B and D with respect to C or P.
We cannot find why he failed.
P
A is not sufficient.
B
A

Using statement B,
PD
K
PC =
3K
3
Let PD be 3k
3k
So PC =
=k
D
C
3
Let x be the side of the square.
We known that PC or PD must lie between x and
since PD (or PC) Diagonal of the square.
So PD/PC lies between 1 / 2 and

2 x,

2 . It can never

Suppose Mohan invested Rs.x in alternative 1 and Rs.y in


alternative 2.
If there was a rise in the market his return (say R1)
5
8
x +
y
=
100
100
If there was a fall in the market his return (Say R2)
4
10
=
x
y
100
100
5y
when x = 0 and it
100
decreases as x increases. R2 has its minimum value of
4y
when x = 0 and it increases as x increases.

100
The assured return (i.e. the smaller of R1 and R2) has its
maximum value when R1 = R2, i.e. when
8 x 5 y
10 x
4
y
+
=

100 100 100 100


y = 2x
2x
His return from either alternative = Rs.
.
100
His total investment = Rs.3x.
2
his return percentage =
%
3

R1 has its maximum value of

2
%.
3
Choice (3)

1.

The greatest assured return for Mohan is

2.

y = 2x gives the maximum assured return percentage of


2
1
1
%.
=
.
3
2
2
Choice (3) follows.
Choice (3)

Solutions for questions 3 to 8:


3.

Let the initial price of coffee be Rs. 10 per kg. Let the
initial consumption be 10 kg. Initial expenditure = Rs.100.
New price = 10(1.4) = 14.
New expenditure = 100(1.12) = 112
112
New consumption =
= 8.
14
Percentage by which Raj must decrease his
10 8
consumption =
Choice (C)
100 = 20%.
10

4.

Let the populations of X, Y and Z in 2008 be 300, 500


and 600 respectively.
Populations of X, Y and Z in 2009 were 300(1.1) = 330;
500(1.125) = 562.50 and 600(1.15) = 690 respectively.
Total population in 2008 = 1400
Total population in 2008 = 1582.50
182.50
Percentage increase =
100 13%.
1400
Choice (C)

5.

Let the cost of each ball pen be Rs.x.


3600
3600

= 90.
x
20

x 1

100

19

1
5
= 90
3600
4x x
900

= 90
x
x = 10.
6.

7.

8.

(2) (1)

Required amount = P +

Solutions for questions 1 to 7:


1.

16

? (say x)

16

16

16

4.

There are 3 separate quantities that we have to keep


track of - the rate of doing work, the time and the
amount paid. We can tabulate the data as shown
below.
A
B
C
D
Rate
4
3
2
3
(The first statement gives the ratio of the rates for A
and B, the second for C and D and the third for B and D.).
The joint rate for AB is 7 units/day. They are paid
`1680. A's share would be

5.

16
Difference = P
= 384.
100
Choice (B)

Let the loan amount be Rs. P. Let the rate of interest be


R% p.a.
9PR
P+
(1) and
= 90,000
100
12PR
P+
= 105,000 (2)
100

4
(1680 ) = `960
7

If all 4 together complete the same work, A's share


would be

Let the sum be Rs.P.

Choice (D)

As the total man hours required to complete the job is


constant, we have
1(50)(20)(9) = 1(50)(x)(9) + 1(45)(12.5)(8)
9000 = 450x + 4500
x = 10.
Choice (D)

Solutions for questions 1 and 2:

2.

64

3.

4
(100 + P) = 27.5
5

384
.10000 = 15000.
256

8 . 64 16 . 16 . x
=
8
16
x=4

Simple Interest Compound Interest

P=

100 + P = 137.5 P = 37.5.


Therefore, he sold the puppy to his neighbour at 37.5%
profit.
Choice (B)

1.

M1H1 M2H2
=
W1
W2

= 27.5
(100 + P)

Choice (C)

Total
Number
Number number Wall length
Men (M) of hrs 1
of days of hours (in m) (W)
day
(H)

So, SP = 100 + P. Therefore CP for the neighbour = 100


+P

4
(100 + P)
5
27.5
Now the part refunded to Pradeep =
(100)
100

Let the required time be x minutes.


Time taken to fill the tank = 50 minutes.
1
1
1

=
x 200 50
x = 40.

2.

Let the profit percent be P %, the profit is P.

Selling at 20% loss implies selling at

Choice (B)

Time and Work

Let the monthly incomes of A, B and C be Rs. 6x, Rs.7x


and Rs.8x respectively.
Let the monthly expenditures of A, B and C be Rs.5y,
Rs.6y and Rs.10y respectively.
Monthly savings of A, B and C are Rs.(6x 5y),
Rs.(7x 6y) and Rs.(8x 10y) respectively.
37.5
(8x ) = 3 (8x ) = 3x
8x 10y =
8
100
5x = 10y
x = 2y.
6x 5y = 7y and 7x 6y = 8y.
7y
(100)% = 87.5%.
Choice (D)
Required % =
8y

P
100= P CP=100
CP

PR
= 5000
100

4PR
100
= 90000 4(5000) = Rs.65000.

Choice (B)

Let the cost price of each mango for Ashok be Rs.x.


2040
the selling price of each mango = Rs.
96
= Rs.21.25.
21.25
= 25.
As loss is 15%, CP per mango =
0.85
If he makes 30% gain, then the SP of each mango
1.30
(25) = Rs.32.50
=
100
2600
Required number =
= 80.
Choice (A)
32.50

3PR
= 15,000
100

4
(1680 ) = `560
12

Choice(C)

Let the number of units done by each man and each


woman in a day be M and W respectively.
(4 M + 12 W) 120 = 4 M (120 3)
4M + 12 W = 12 M
8 M = 12 W
2M=3W
Let 2 M = 3 W = 6
M = 3 and W = 2

33 1
3 (3 )
100
50
= 4 units and each woman will do 2+
(2)
100
Now, in winter each man will do 3 +

20

a < b

= 3 units
i.e. each man and each woman do 4 units and 3 units
respectively in one day.
The original work
= (4 M + 12 W)120 = (4 3 + 12 2) 120 units
Two times the work = (36) 120 2 units
Now in winter 'a' men and 'b' women take
180 days
(4a + 3b) 180 = 36 120 2 4a + 3b = 48
We need to find the number of nonnegative integral
solutions to 4a + 3b = 48
Dividing both sides of equation (1) by 3, we get

a
= x. x < 1.
b
25
1
x+
=
x
12
25
x2
x +1=0
12
Let

12x2 25x + 12 = 0
(3x 4) (4x 3) = 0

4
3
3
or
As x < 1, x =
3
4
4
a 3
or
=
b 4

x=

3a
48
a
3b
+
+
=
3
3
3
3
a
= 16 b a
3

say a = 3k, b = 4k
The total work = 12 (a + b) = 12 (7)k
A's rate = a = 3k

a
16 b a is an integer, so
must also be an integer
3
a
= K where K is an integer
Let
3

Alternative Solution:
Let A and B take a days and b days respectively to
complete the given piece of work. Given that

a = 3K
Now substituting a = 3 K in equation (1), we get
b=

48 4(3K )
b = 16 4 K
3

1 1
1
a b
1
--- (1) and
+ =
+ = 24
a b 12
2 2
2

Substituting K = 0, 1, 2, 3 and 4, we get the values of a


and b satisfying the given equation.
k
a
b

0
0
16

1
3
12

2
6
8

3
9
4

a + b = 49---(2)
Solving (1) and (2), we get (a, b) = (28, 21) or (21, 28).
But as a > b, a = 28
Choice (A)

4
12
0

Venn diagrams

Thus the number of ordered pairs (a, b) satisfying the


given equation is 5.
Choice (D)
6.

12(7)
or 28 days
3

A would take

Solutions for questions 1 to 3:


1.

T (30%)

Let the daily wage of B be 5K


The daily wage of A is 5K

40
(5K) = 3K
100

a%

Thus the total wage of A and B in a day = 5K + 3K = 8K


Together they completed the work in 72 days.
So the total money received by them = 72 (8K).
If this entire amount is to be earned by A alone at the
rate of 3k per day, it would take him

72(8K )
= 192 days.
3K
7.

b%

Let the strength of the class be T.


Number of students who like both tea and coffee
a
20 30
(T ) =
=

T
100
100 100

Choice (D)

Let the amount of work done per day by A and B be a, b


respectively.
The agents, their rates and the time taken are tabulated
below
Agent
Rate
Time

A
a

B
b

a+b

12

a+b

12

Work

AB
a+b
12

a = 6.
Required percentage = b% = 100% (Percentage of
students who like either tea or coffee) = 100% (30% +
40% 6%) = 36%.
Choice (D)
2.

Son
Star

12(a+b)

180
c

1
a+b
a+b
+ 6
= 24
2
a
b

1
b a
or, 6 + = 12
2
a
b

b a 25
or,
+ =
a b 12
A is slower

C (40%)

210

90

150
Zee
a+ 90 + b + 210 340 [since at most 340 residents
watch Star Plus]
a + b 40 (1)
180 + 90 + a +b + c + 210 = 540 [ Q 540 residents watch
at least one of Sony TV and Star Plus]

21

a + b + c = 60.
(1) c 20.
Required number = Minimum value of c = 20.
Choice (C)
3.

2.

P
10m

Let the number of boys and the number of girls be


b and g respectively.
Boys
Girls
Number of persons liking cricket 0.6 b
0.5 g
Number of persons liking volleyball 0.45 b 0.55 g
0.6 b + 0.5 g = 7 + 0.45 b + 0.55 g
0.15 b = 7 + 0.05 g i.e., 3b = 140 + g (1)
0 .6 b 0 .5 g
0.45 b 0.55 g

45
R

(0.6 b 0.5 g) 16
=
.
(0.45 b 0.55 g) 5

If

3.

b 3
= , (1)
g 2

Trigonometry

1.

Q1

2 PQ
3

2(5 2 )
3

10 6
m.
3

If p, q, r are any 3 real numbers, pq + qr + rp p2 + q2 + r2.


If p, q, r satisfy the condition that the sum of any two
exceeds the third (or equivalently the sum of the two
smaller numbers exceeds the greatest), then all of them
have to be positive.
p + q > r pr + qr > r2.
q + r > p pq + pr > p2.
p + r > q pq + qr > q2.
Adding these and simplifying, we get
p2 + q2 + r2 2 (pq + qr + rp)
1
pq + qr + rp
< 2

2
p + q2 + r 2

Choice (D)

243
729
2187
6561

19683
59049
177147

We can check that 177147 59049 is 118090.


33x + 2 92x 1 = 311 95.
If 3x + 2 = 11 (as x = 3) then 2x 1 = 5.
x = 3 and 6x + x6 = 63 + 36 = 216 + 729 = 945.

30

P1

As the LHS consists of power of 3s, RHS should also


be expressed in the form of power of 3. We need to list
the powers of 3 until we get numbers greater than
118098.
3
9
27
81

30

Indices, Logarithms, Surds

Again R and S are equidistant from Q [ Q QRQ1


= QSQ1 = ] Thus QR = QS

PQ
tan 60 0

Solutions for questions 1 to 7:

P1

30

1
1
< cos and hence
< cos 1
2
2
i.e., 0 < 60.
cant be 80

Let Q Q1 and P P1 be the towers T1 and T2. It is given


that QRQ1, and QSQ1 are both '' where R and S
are the two points on the ground. Again P1 Q = R P1
= SP1 = 30 feet, where R, P1 and S are collinear.

PQ
tan 60 0

i.e.,

Solutions for questions 1 to 3:

Choice (B)

b = 60 and g = 40
b 113
=
, (1)
If
g 102
then 60% of b and 55% of g are not integers.
b = 60 and g = 40. Hence, the required strength
= 100.
Choice (B)

1.

60
Q

PQ.
= sin 45
PR
1
= 5 2m.
PQ = 10.
2
Required distance = ST
= SQ + QT.

16
.
5

(3 b 2.5 g) = 7.2 b 8.8 g.


3b 2.5 g = 7.2 b 8.8 g or 3b + 2.5 g = 7.2 b 8.8 g
6.3 g = 4.2 b or 11.3 g = 10.2 b
b 113
b 3
i.e.,
or
=
.
=
g 102
g 2

60

30

Alternative Method:
118098 can be expressed as
= 9 13122 = 9 9 1458
= 92 9 162
118098 = 93 9 18
= 95 2
= 310 2
= 310[3 1] = 311 310

In the above RQS is isosceles.


Thus QP1S = QP1R = 90
Now, in the QRS, QP1 is the median to RS which is
1/2 RS. This is possible only when RQS = 90.
Thus the area of QRS = 1/2 (RS) (QP1) = 1/2 (60)
(30) = 900 ft2.
Choice (D)

2.

Choice (A)

Consider the following expression:


1
1
=
(n + 1) n + n n + 1 n(n + 1) n + 1 + n

)
22

n +1 n

n(n + 1)

17 16 + 16 17
1

18 17 + 17 18

n +1

16

1
17

The required sum is

+....

120
1

16

17

7.

120

Given that t = 3( log3 a )2 9 log3 a + 9 and at = 27

1
1
16

x3-3x2+3x-1=0 (x1)3=0 x = 1.
There is only one distinct value for x (or log3 a ) or for t
(as tx = 3)
Choice (A)

121

3
= 3x29x+9 3x3-9x2+9x-3=0
x

17

17

Modulus / Inequalities

18

Solutions for questions 1 to 10:

1
121

1.

7
1
1

=
=
4
11
44
121

Choice (B)

Given (x2 + 4x 32) (x2 + 2x 8) > 0


(x + 8) (x 4) (x + 4) (x 2) > 0
The critical points are 8, 4, 2, 4.

3.

1
(1)
x+ =3
x
Squaring on both sides, we get
1
x2 +
(2)
=7
x2

Combing both sides, we get x6 +


2
1
= 73
x +
x
x2
1
x6 + 6 + 3(7) = 343
x
1
x6 + 6 = 322
x

1
x6

+ 3x2

2.

5.

9602 4801
=
970
485

3.

As log77, log7(4x + 5), log7 (4x + 1 1) are in AP, the


3 numbers are in GP, i.e.,
log77 + log7(4x + 1 1) = 2log7(4x + 5)
= log7 7(4x + 1 1) = log7(4x + 5)2
7 (4x + 1 1) = (4x + 5)2

4x 2 9x + 8
x 2 5x 6

> 3.

4 x 2 9 x + 8 3( x 2 5 x 6)
x 2 5x 6

>0

( x 2 + 6 x + 26)( x 2 5 x 6)

>0
( x 2 5 x 6)2
as x2 + 6x + 26 is always positive
2
x 5x 6 > 0
(x 6) (x + 1) > 0
x < 1 or x > 6 (, 1) (6, ).

Alternative Method:
Substitutes the choices, only Choice (1) satisfies.
Choice (A)
6.

a2 b2
( a b)

b
a

(a b)(a 2 + ab + b 2 ) ab(a b)
0
ab
ab
0 ( a2 + b2 is always positive)

ab
(a b) ab 0
(a b) 0 and ab < 0 ( as ab 0) or (a b) 0
and ab > 0
a b and ab < 0 or a b and ab > 0. Choice (C)

Choice (D)

The two bases in the logs are 2 and 4. Let x = 22p = 4P


Applying log on both sides
log4x = P and log2 log4x = log2P.
log2x = log24p = log222p = 2p.
log 2 2P
log4log2x = log42P =
2
log 2 2p
Given log2P =
2
2 log P = log 2 + log P
P = 2 x = 24 = 16.

Given

(a 3 b 3 ) (a b)(ab)
0
ab

a + b = 10 and ab = 1
a3 +b3 = (a + b)3 3ab(a + b) = 103 3(10) = 970
a4 +b4 = (a2.+.b2)2 2a2b2 = [(a + b)2 2ab]2 2a2b2
(102 2)2 2
= 982 2 = 9602

a3 + b3

V
4

a3 b3
(a b) 0
ab

Choice (C)

a=52 6 ,b=5+2 6

a4 + b4

V
2

When x = 0; the equation is true.


The values of x in first, third and fifth region are
satisfied.
The values in second, fourth region are not satisfied.
The number of values of x that the inequation do not
satisfy are x [ 8, 4] [2, 4] ie 8 values.
Choice (B)

4.

Choice (C)

t = 3x2-9x+9 (where x = log3 a ) and tx = 3

18

121 120 + 120 121

Let 4x = y.
28 y 7 = y2 + 10y + 25
y2 18y + 32 = 0
x
x
y = 16 or y = 2, i.e., 16 = 4 or 2 = 4
x = 2 or x =

4.

Choice (D)

Given x1 x2 x3 x4 x5 x6 = 1
If (x1 + 4) (x2 + 4) (x3 + 4) (x4 + 4) (x5 + 4) (x6 + 4) is
minimum when x1 = x2 = x3 = x4 = x5 = x6 = 1.
The minimum value is 5 5 5 5 5 5 = 15625.
Choice (A)

23

5.

The lower bound of S can be estimated as follows

If x is 7, | x-5| + | x + 3| + | x + 7| =26.
At x = 7, E = 16
At x = 5, E = 20
For x > 5, if x is increased by 1, E increased by 3.
For x < 7, if x is decreased by 1, E increased by 3.
At x = 10, E = 16 + |10 (7)| = 3 = 16 + 3(C) = 25
At x = 6, E = 20 + |7 6|3 = 23
At x = 7, E = 10 + 2 (3) = 26.
If x lies between 7 and 5, |x + 7| + |x 5| is 12.
For |x 5| + |x + 3| + |x + 7| to be greater than 14,
|x + 3| must be greater than 2.
Thus x cannot lie in the range 5 x 1
Therefore x can take the values 9, 8, -7, -6 and 0, 1,
2, 3, 4, 5 and 6 i.e. x can take 11 integral values
Choice (B)

1 1 1 1 1 1 1 1 1
1
1
1 1
1+ + + + + + + + + +...+ +...+ + +...+
2 3 4 5 6 7 8 9 10
16
512
257 258

Lower bound can be considered as


1 1 1 1 1 1 1 1 1
1
1
1
1
1+ + + + + + + + + +...+ +...+
+
+...+
2 4 4 8 8 8 8 16 16
16
512
512
512

2 Times

4 Times

8 Times

256 Times

1 1 1 1 1 1 1 1 1
= 1 + + + + + + + + + = 5.5
2 2 2 2 2 2 2 2 2
Again the upper bound of S can be estimated as follows
1 1 1
1 1
1 1 1 1 1 1 1 1 1 1
1+ + + + + + + + + + +..+ +..+ + +...+ +
2 3 4 5 6 7 8/ 9 10 11 15 256 257 511
512
1 1
1 1 1 1 1 1 1 1 1 1 1 1 1
1+ + + + + + + + + + +..+ +..+ + +...+ +
2 2 4 4 4 4 8 8 8 8 8 256 256 256 512
2 Times

4 Times

8 Times

9.

1
= 1 + 1 + 1 + 1 + 1 + 1 + 1 + 1 + 1[
is very
512

6.

P ). The sum of the four numbers

would be 44 P . Thus, as long as 4 4 P P , it would


be possible to find the required numbers.
4

that

appear

in

the

inequality

are

where each quantity in the modulus sign is positive or


negative, i..e, we have to identify the lines across which
the sign changes.
As x appears, we have to take the equation x = 0

Choice (B)

If the product of 4 numbers is the constant P, the sum S


would be minimum when all the four are equal (and
hence equal to

terms

x , y and x y . We have to identify the regions

256 Times

small and can be ignored.] = 9


Therefore 5.5 < S < 9.

The

As y appears, we have to take the equation y = 0


As x y appears, we have to take the equation

xy =0
We get the 6 regions as shown in the figure.

4 P =P4 =P
3
P = 4 4 4 1 .6 = 6 .4

( )

If P < 6.4, 4 4 P > P ie. the minimum value of the sum


would be more than P and it would not be possible to
find 4 numbers of the required kind.
If P > 6.4, 4 4 p < P i.e, the minimum value of the sum
would be less than P and it would be possible to find 4
numbers of the required kind.
While 7 is a possible value for P, 4, 5, 6, are not.
There is only 1 value among the 4 given, which is
possible for P.
Choice (B)
7.

x + y + z = 125
One of x, y and z is 5
Regarding the other two, there are two possibilities.
Either both are 0 or the two are of the form +n and n,
where n is an integer other than 0.
a2 + b2 + c2 = 16 One of a, b, c is 4 and the other two
are 0.
ax + by + cz = 20
4(5) + 0(n) + 0(n) = 20.

a+b+c
a+b+c
=
4 x 2 + y 2 + z2
4 x2 + y2 + z2
a+b+c
For n = 0, 1, 2, 3, 4 the values of
4 x 2 + y 2 + z2
4
1
1
1
1
1
=
are
,
,
,
,
2
25
27
33
43
57
4 25 + 2x

Now

D
F
E
The sign of the 3 quantities in the six regions are
tabulated below.
A
B
C
D
E
F

8.

y
+
+
+

xy
+

+
+

We have to simplify the given inequality (get rid of the


modulus signs) separately in the 6 regions.
(A)

x + y + xy
=x+y+xy 2x1

(B)

x + y + xy
=x+y+yx2y1

(C)

respectively.
it cannot be

x
+
+

x + y + xy
=x+y+yx2yx1

1
.
49

Choice (D)

-10

-7

-3

x a is the distance of x from a on the number line


Let = | x 5 | + | x + 3 | + | x + 7| = 25.

(D)

x + y + xy

= x y + y x 2 x 1
(E) |x| + |y| + |x y|
= x y + x y 2
y 1
(F)

x + y + xy

=xy+xy2xy1
The enclosed area = 3

Choice (A)

24

10. It is given that (p + q + r)2 = 3(pq + qr + pr)


p2 + q2 + r2 + 2pq + 2qr + 2pr
= 3pq + 3qr + 3pr
2
2
2
p + q + r = pq + qr + pr
Multiplying both sides by 2, we get
2p2 + 2q2 + 2r2 = 2pq + 2qr + 2pr
2p2 + 2q2 + 2r2 2pq 2qr 2pr = 0
(p q)2 + (q r)2 + (r p)2 = 0
The sum of three perfect squares will be zero only when
each of them is zero.
p q = 0, q r = 0 and r p = 0
p=q,q=r,r=p
Thus p = q = r
It must be an equilateral triangle.
Choice (A)

3.

As per the definition, Div(N) always takes a single-digit


number for its value.
We see that any number and the sum of the digits of
that number have the same 9s remainder (remainder
rule for 9). In the process of adding up the digit of a
given number, repeatedly, the 9s remainder doesnt
change.
The number N for which Div(N) = 9, are the multiples
of 9.
600
There are
or 66 multiples of 9 less than 600.
9
Choice (3)

Operator Based Questions

4.

T1 =

Solutions for questions 1 to 3:


1.

2.

t2 1+

t3 =

x
y
+
y
x

y x
+ .
x y

5.

3891
4179
=
36
36

1
3

a * b = a + b + ab
a b = ab(a + b)
3 * 4 = 3 + 4 + 3 (4) = 19
4 5 = 4 (5) (4 + 5) = 180
19 18 = 19 (180) (199)

= 1+

1
32

Choice (C)

On replacing two numbers say p and q by p + q 1,


the following are the consequences.
(i) the number of numbers on the board decreases by 1.
(ii) the sum of the numbers on the board decreases by 1.
The above consequences mean that after the
procedure is carried out 49 times, there is a single
number whose value is 49 less than the original sum.
(50) (51) 49 = 1275 49
final number =
2
= 1226.
Choice (5)
73024 = (74)756 = (2401)756
(74) raised to the power of any natural number must
end with 01
(74)756 must end with 01.
Choice (4)

1 1
+ =
4 9

36 + 9 + 4
7
=
6
36

8
1
=3
3
3
1

42

144 + 16 + 9
3242

13
12

Choice (2)

We can arrange the elements of A in 2 columns


(1 and 2) as shown below. The numbers that we get by
increasing each odd number by 3 and increasing each
even number by 1 are shown in the column 3 and 4
respectively.
1
2
3
4
3
4
6
5
5
6
8
7
2N + 2

2N + 4

2N + 3

P is the average of the resulting odd numbers (i.e.


column 4). Q is the average of the resulting even
numbers (i.e. column 3). We see that PQ = 1.
Choice (3)
6.

1
1 3
=

x 18 y
1 3
1
+ =
x y 18

Solutions for questions 1 to 43:

2.

1+

9
3
=
2
4

3
1
=2
2
2

2N + 1

Numbers

1.

32
13 15
1
+
=
=4
4
12
12
4
1
Similarly s2005 = 2006
2006

Choice (C)
3.

22

s3 =

Choice (A)

1
6
1
1
1

= 8 + 3 36 +

+
= 8.6
+3
6@
1 6
36
6
6

6
6
=8+

s2 = t1 + t2 =

3
3

Given x @ y = y 4 x + + x 4 y +
y
x

3x
3y
= 8xy +
+ 4xy +
y
x

1
12

s1 = t1 =

a b = 4a + 3b + 7ab
a a= 4a + 3a + 7a2
= 7a(1 + a)
b b = 4b + 3b + 7b2
= 7b (1 + b)
Given a a < b b
7a (1 + a) < 7b (1 + b)
a + a2 b b 2 < 0
(a b) + (a2 b2) < 0
(a b) (a + b + 1) < 0.

= 4xy +

1+

18(y + 3x) = xy
xy 18y 54x + (18) (54) = (18) (54)
(x 18) (y 54) = 35 22
x is odd and less than 100.
x 18 is odd and less than 82.
x 18 = 34, 33, 32,31, or 30.

Choice (5)

7.

There are x players in each team and N pairs of teams


have a common player.
total number of players in the 2N + 1 teams
= x(2N + 1) N = N(2x 1) + x.
Choice (2)

8.

Sum of the odd digits of the number = sum of its even


digits = A + B.
It is divisible by 11.
It is a perfect square.
it must be divisible by 112 i.e., 121 (1)
Its last two digits are equal.

25

The tens digit of any odd perfect square is even.


Therefore, AABB is the square of an even multiple of
11.
AABB is 442 (i.e. 1764), 662 (i.e. 4356) or 882
(7744).
AABB = 882
= 7744. So, only 1 possibility.
Choice (4)
9.

n(n!) = [(n + 1) 1]n! = (n + 1)! n!


3(3!) = 4! 3!
4(4!) = 5! 4!

15(15!) = 16! 15!


M = 16! 3!
and M 15 = 16! 6 15 = 16! 21
The divisor is 14! 2
Consider (14! 2) (15) (16). It is
16! 480
The dividend is 16! 21 = 16! 480 + 459
16! 21 459
= R
= 459
R
Choice (C)
14! 2 14! 2

10. The divisors and


tabulated below

corresponding

Divisor
Remainder

5
3

6
4

7
5

remainders

are

8
6

1
+ .
32
1
1
1
Let y = 2 + 2 + 2 + .......
2
4
6
22

1
1
1
1
1 +
+
+ ..... =
(1 + x)
4
22
22 3 2

1
1
1
+
+
+ .. or x y.
we need
32 52 72
1
3x 1
(1 + x) =
xy=x
4
4

Choice (A)

12. It is convenient to consider a general expression of the

form

E=

an + bn
an + 1 bn + 1
n

a +b

49 80 42 80
49 79 + 42 79

(a b) (a n + a n + 1b + .... + b n )
an + bn

> 49 42 i.e. 7

15. Consider the choices.

(A) (7 + 4 3 ) 50 = (7 4 3 )50 = 750 50C1 749 ( 4 3 ) +


C2 748 ( 4 3 )2
The expression in A is double of this and hence
irrational.

50

Choice (C) is 7 + 4 3 , which is irrational.


Choice (D)
16. In n is odd, an + bn is divisible by a + b.
If there is odd factor in the index n, an + bn cant be
prime for example.
296 + 1 = (232)3 + 13 is divisible by 232 + 1
2160 + 1 = (232)5 + 15 is divisible by 232 + 1
270 + 1 = (210)7 + 17 is divisible by 210 + 1 Choice (D)
17. The first sheet provides pages 1, 2, 199, 200. The total
is 402.
The second sheet provides pages 3, 4, 197, 198. The
total is 402
The total on any sheet is constant, irrespective of n.
Choice (D)

an + 1 bn + 1

14. Let a = c
2c + 5b

= (c 2b 5 )1 / 7
7
or the AM of c, c, b, b, b, b, b is equal to the GM of the
7 numbers.
c +b
All the numbers are equal, i.e. b = c and x =
=b=c
2
2
5
2(x a) + 5(x b) = 0.
Choice (D)

(B) (7 4 3 ) 50 = (7 + 4 3 )50 =750+50C1 749 ( 4 3 ) + .


The expression in B is double of this and hence
irrational.

The required numbers are of the form [LCM (5, 6, 7, 8)]k 2


or 840k 2. The required numbers are
840(2) 2
840(3) 2

840(11) 2
There are 10 such 4-digit numbers.
Choice (B)
11. x =

i.e. instead of A itself (or the individual digits of A) we


consider B = 7 + 14 + 21 + + 189 + 196 = 7(1 + 2 + 3
+ + 27 + 28) = 7(14)(29)
The 9s remainder of the product of several numbers is
the product of the 9s remainders of the numbers.
The required remainder is the remainder of 7(5) (2)
or 7.
Choice (D)

>ab

Choice (D)

13. To obtain the 9s remainder of any number, we can


proceed in a number of ways.
At one extreme, we can consider the given number
itself as a single quantity.
As the other extreme, we can focus on each individual
digit and add them all up.
As an intermediate approach, we can fragment the
number in any convenient way and consider the
fragments.
In the given question, the third approach is the most
convenient.

18. u + v + w + x + y = 15
We need the max values of uvx + uvy + uwx + uwy
= (uv + uw) (x + y) = u(v + w) (x + y). This occurs when
u = v + w = x + y = 5 and it is 5(5) (5).
Choice (A)

1301
19. Rem
= 20 1
21
Let the given expression be E.
E
Rem = (1)10 (1) = 2
21

Choice (B)

20. 23 < a < b < 40 < c < d < 50


There are 16 number between 23 and 40, therefore 16C2
ways of selecting (a, b)
There are 9 numbers between 40 and 50, therefore 9C2
ways of selecting (c, d)
16(15)
9(8)
16
C2 =
= 120 and 9C2 =
= 36
2
2
The number of values of (a, b, c, d) is 120(36)
= 4320
Choice (B)
21. N = 15a + 7 .. (1)
= 21b + 10 (2)
= 35c + 17 (3)
(1), (2) 15a 21b = 3

26

5a 7b = 1
5a 5b = 2b + 1
2b = 5a1 + 4
a1 = 2a2
a2 = 0 a1 = 0 b = 2 a = 3. (a, b) = (3, 2),
(10, 7) (17, 12) etc.
N = 15a + 7 = 15(3) + 7
or 15(10) + 7 etc
i.e. N = K LCM (15, 21) + 52
= 105k + 52
when such a number is divided by 35, the remainder is
17.
The numbers of the required kind leave a remainder
of 52 when divided by 105.
Choice (C)
22. Since there is no restriction that the digits are distinct,
we can consider all the multiples of 6.
99996 10002
Number of 5digit multiples of 6 =
+1
6
89994
=
+ 1 = 15000. These include all the multiples of
6
12.
99996 10008
Number of 5digit multiples of 12 =
+1
12
= 7500
Number of 5-digit multiples of 6 that are not multiples
of 12 = 15000 7500 = 7500
Choice (A)
12

24

12

12

23. 4 1 = 2 1 = (2 1) (2 + 1)
= (26 1) (26 + 1) (212 + 1)
= 7(9) (65) (4097) = 32(5) (7) (13) (17) (241)
412 1 is divisible by 5, 63 and 255.
Choice (D)
24. 243243 = 243 1001 = 35 7 11 13
= 21(34 11 13)
The number of factors of 243243 that are divisible by 21
243243
= Number of factors of
21
= Number of factors of 34 11 13
= (4 + 1) (1 + 1) (1 + 1) = 20
Choice (A)
11

11

11

= 33 + 11 + 3 + 1 = 48
Highest power of 2 in 30!

30 30 30 30
+ 2+ 3+ 4
2 2 2 2

= 15 + 7 + 3 + 1 = 26

30 30 30
+ 2+ 3
3 3 3

Highest power of 3 in 30! =

= 10 + 3 + 1 = 14
We can tabulate these results as shown below.
Nos

1 to 30

1 to 100

31 to 100

No of 2s

26

97

97 26 = 71

No of 3s

14

48

48 14 = 34

In N there are 71 twos and 34 threes. As N has 71

71
fours i.e., 35 fours.
2

twos, it has

The highest power of 12 that divides N is 1234.


Choice (B)
28. The number of coprimes to ab less than ab is given by

1
1

ab1 1 = (a -1) (b 1)
a b
Therefore n can take (a 1) (b 1) values. The sum of
these values =

(a 1) (b 1)ab
2

Choice (D)

29. Among the elements of set A, there are 5 multiples of 7


i.e, 7, 28, 49, 70 and 91.
The GCD of the elements is 7, when all the elements
are selected from these 5 numbers.
Therefore, the number of proper subsets of A satisfying
the given condition is 25 1 i.e, 31. [Each of the above
numbers is either present or not present. So for each of
them, there are 2 ways of constructing the subset. We
subtract 1 to exclude the null set].
Choice (B)

11

25. 49 + 52 is divisible by 101 and 50 + 51 is divisible


by 101.
It can be noted that 4911 + 5011 + 5111 + 5211 is an even
number. Hence it is divisible 202.
Hence the remainder when 4911 + 5011 + 5111 + 5211 is
divided by 202 is 0.
Choice (A)
26. 256! has (128 + 64 + 32 + 16 + 8 + 4 + 2 + 1) i.e. 255 twos,
(85 + 28 + 9 + 3 + 1) i.e. 126 threes and (51 + 10 + 2) i.e. 63
fives as its factors. When all the zeros at the end of 256!
are knocked off, there are no fives left and there are (255
63) i.e. 192 twos left. To get a 12, 2 twos and 1 three
are needed. We have 126 threes and 192 twos
192
available. So, a maximum of
i.e.. 96 twelves are
2
available.
The index of the highest power of 12 in Q is 96 i.e.
1296 divides Q but 1297 does not.
Choice (C)
27. We want the highest power of 12 in N. This equals
(Highest power of 12 in 100!) (Highest power of 12 in
30!)
Highest power of 2 in 100!

30. The only way to express 9 as the product of 4 distinct


integers is (3)(1)(1)(3)
(n p)(n q)(n r) (n s) = 9
= (3)(1) (1)(3) (I)
As the 4 integers p, q, r, s are such that the equation
has solutions, the differences between adjacent
integers when the 4 are arranged in ascending order
are 2, 2, and 2. For example, if p > q > r > s,
np<nq<nr<ns
(n p) = 3
(n r) = 1
(n q) = 1
(n s) = 3
n=p3=q1=r+1=s+3
Thus, for any given (p, q, r, s), there is only one value of
n which would satisfy ().
Choice (B)

33333 44444

31. Rem

33333 44444

= Rem

7( 4762) 1

44444

100 100 100 100 100 100


=
+ 2 + 3 + 4 + 5 + 6
2 2 2 2 2 2

= 50 + 25 + 12 + 6 + 3 + 1 = 97
Highest power of 3 in 100! =

Now, Rem 7

100 100 100 100


3 + 2 + 3 + 4

3 3 3

Again 44444 = 7(6349) + 1

4762
1
7

44444

7k + 1
=1
7

= Rem

27

44444 33333

Rem

6349 33333

= Rem 7
+ 1

7k + 1
Rem
=1
7

33333 44444 + 44444 33333

Thus Rem

=1+1=2

Choice (B)

32. 4 + 44 + 444+ 4444+ ... + 444 . . . . 50 times, when


divided by 9 will leave the same remainder as
4+8+12+...(50 terms) is divided by 9. Sum of the digits
in a number and the number when divided by 9 leave
the same remainder.
4+4=8
4 + 4 + 4 = 12
4 + 4 + 4...50 times = 200)
Therefore, we need to find the remainder when
4 + 8 + 12 + ...+ 200 is divided by 9

Now, 4 + 8 + 12...+ 200 =

50
(204) = 5100
2

5100 when divided by 9 will leave a remainder of 6.


Choice (C)
32.

1 1 1
+ = , ..1
a b 6
6a + 6b + ab
ab + 6a + 6b = 0
Adding 36 to both sides, we get
ab 6a + 6b + 36 = 36..
(a 6) (b 6) = 36 ---------- (A)
Now, 36 can be written as a product of two integers in
the following 10 ways:
When both integers are
When both integers are
negative
positive
( 1) ( 36)
1 36
2 18
(2) ( 18)
( 3) ( 12)
3 12
( 4) ( 9)
49
( 6) ( 6)
66
The number of ordered pairs (a, b) which satisfy the
equation is 17.
(6) x (6) doesnt yield a solution for (a,b).
( (a-b, b-6) = (-6, -6)
(a,b) = (0,0) which doesnt satisfy (D)
6 x 6 gives one value of (a,b). Each of the other 8
values of {a-6, b-6} gives two values of (a,b)
Therefore, the number of solutions = 1 + 2(8)=17
Choice (C)

33. Let the two integers be a and b. It is given that


a2 b2 = 420 ;
(a + b) (a b) = 420
Therefore (a + b) as well as (a b) are factors of 420.
420 = 22 x (3) x (5) x (7).
The number of factors of 420
= (2 + 1) (1 + 1) (1 + 1) (1 + 1) = 24
The different ways in which 420 can be expressed as a
product of two numbers and the corresponding values
of a, b are tabulated below.

S.No.
1.
2.
3.
4.
5.
6.
7.
8.
9.
10.
11.
12.

420 1
210 2
140 3
105x4
84 5
70 6
60 7
42 10
35 12
30 14
28 15
21 15

106

104

38

32

26

16

22

We set (a + b) equal to one factor and (a b) equal to


the other. But both factors have to be even.
From the above 12 ways of factorizing 420, we will get
integral values of a and b such that a > b only for the
following four ways.
a + b = 210
a + b = 70
a + b = 42
a + b = 30

ab=2
ab=6
a b = 10
a b = 14

We get 4 values of (a,b). By interchanging the numbers


in each pair, we get 4 more. By taking both factors to be
negative, we get 8 more. Therefore the number of pairs
of integers is 16.
We consider one set of 4 values
a + b = 210, a b = 2 (a, b) = (106,104)
a + b = 2 , a b = 210 (a, b) = (106, 104)
(a + b) = 210, a b = 2
(a, b) = ( 106, 104)
(a + b) = 2, (a b)= 210
(a, b) = ( 106, 1040)
Similarly, well get 12 more values for (a, b) i.e. a total of 16
ordered pairs.
Choice (D)
34. The 9-digit numbers must have either no 8 s, three 8 s,
six 8 s or all 8 s. But, as the numbers are even, each of
them must end in 8. Therefore, we cant have zero 8s.
Number of possible values with nine 8's is 1. Number of
possible values with six 8 s is 8 C 3 [the three 9 s can
occupy any 3 of the remaining 8 positions.] Number of
possible values with three 8's is 8 C 6 [The six 9s can
occupy any 6 of the remaining 8 positions] Number of
possible values with zero 8 s is 0.
Total number of such 9 digit even numbers
= 1 + 8 C 3 + 8 C 2 + 0 =1 + 56 + 28 + 0 = 85
Choice (D)
35. Consider the sum of 2 numbers first instead of 4, i.e.
E = (a + b) 5 a5 b5
E = 5 C 1 a4 b + 5 C 2 a 3 b 2 + 5 C 3 a 2 b 3 + 5 C 4 a b 4
Every coefficient is divisible by 5 E is divisible by 5.
We can write (a +b)5 = a5 + b5 + M(5)
Now consider (a + b + c + d)5
(a + b + c + d)5 = a5 + (b + c + d)5 + M1(5)
= a5 + b5 + (c + d)5 + M2(5)
= a5 + b5 + c5 +d5 + M3(5)
E is always divisible by 5.
We can select examples to show that E is not always
divisible by the other numbers expressions.
(A) For 24, we can think of a simple example
Say (a, b, c, d, ) = (1, 1, 1, 1)
E = 45 4 = 1020, which is not an M(8) and
hence not an M(24).
(B) Say (a, b, c, d) = (1, 1, 1, 4)
E = 75 (1 + 1 + 1 + 1024)

28

As 75 is a multiple of 7, or not a but (1 + 1 + 1 +


1024) is not, E itself is not a M(7) multiple of (a + b
+ c + d)
(C) The same example would serve to show that E is
not an M(9)
Choice (D)
36. The last digit of A is the same as the last digit of
1! + 2! + 3! + 4! = 1 + 2 + 6 + 24 or 33 which is 3 since
5!, 6!, 7!,..., 49! all end with 0.
Thus 1! + 2! + 3!+ ... + 49! is not a perfect square, as a
perfect square cannot end in 3. Similarly, the last digit
of B is also 3 and so we can conclude that B also
cannot be a perfect square.
Choice (D)
4

37. 3600 = 2 3 5
The sum of all the factors of 3600

(25 1) (33 1) (53 1)


2 1
3 1
5 1

= (31)(13)(31) = 12493.
The sum of the odd factors of 3600 is the same as the
sum of the factors of the number 32.52 which is
3 1

41. Let the number with the smallest number of digits


having the given property be a1, a2....an-17
a1 a2..an1 7
5
5
As the result of the multiplication would have the
7 transposed, its last digit is an 1 = 5.
(7)(5) = 35. carry over is 3. Tens digit of result =
Last digit of (5)(5) + proceed till one of the digits in the
result is 7.
We find that we get a 7 in the result when it has 6 digits.
The number then = 714285.
A 18 digit number the having given property can be
obtained by having three blocks of the least number
one after another X can be 142857142857142857.
As there is no cannot be determined in the choices, it
follows that x must be 142857142857142857.
Choice (C)
42. The number of numbers less than 252 and coprime to

252 or [(9) (28) = 22327] is 252 1

(3 ) (5 1)
= 403
3 1 5 1
3

Therefore the sum of the even factors of


3600 = 12493 403 = 12090.
Choice (D)
38. Consider the 5digit number abcde
Let a + c + e = U
b+d=T
Given U + T = 41
and U T = 11 or 11
U T has to be odd and can't be 33, 55, etc. nor can it
be 33, 55, etc
U = 26, T = 15
or U = 15, T = 26 (This is not possible as T 18)
U = 26, T = 15
i.e. a, c, e must have values of 9, 9, 8 in any order
and b, d must have values of 6, 9 or 7, 8 in any order.
(a, c, e) = (9, 9, 8), (9, 8, 9) or (8, 9, 9)
and (b, d) = (6, 9), (9, 6), (7, 8) or (8, 7)
There are 3 (4) or 12 possible values for (a, b, c, d, e)
Choice (B)
2 2

39. 36 = 2 3
We can split the twos as 20, 20, 22 or 20, 21, 21
Similarly we can split the threes as 30, 30, 32 or 30, 31, 31
Then we can pair them as follows:
20 2 0 2 2
20 21 21
0
0
2
3
3
30 30 32
3
30 32 30
32 30 30
30 31 31
30 31 31
31 30 31
31 31 30
Thus, we get 8 ways of expressing 36 as a product of 3
positive integers, viz
1 (1) (36)
1 (2) (18)
1 (9) (4)
9 (2) (2)
1 (3) (12)
1 (6) (6)
3 (3) (4)
3 (2) (6)
Choice (A)
40. Root (0) = 0
Root (1) = 1
Root (2) = 2
From the given example and from the above values it

1
1
1
1 1
2 3
4

1 2 6
= 252 = 72
2 3 7
The number of numbers coprime to 252, in different
ranges are tabulated below
No.of
coprimes

Range

72

1 to 252

72

253 to 504

72

505 to 756

72

707 to 1008

1009 to 1108 or 1 to 100

The required number is 4 (72) + X, but we have to find X.


Every composite number less than 100 has a prime
factor less than 10
The numbers less than 100 and coprime to 2, 3, 7 are
1, all the primes (other than 2, 3, 7) and all the
composites which have only 5 as a factor or 5 and a
prime number more than 7 as a factor i.e. 1, the 25 3
primes and the numbers 52, 5 (11), 5 (13), 5 (17), 5
(19).
Thus, 1 + 22 + 5 or 28 numbers are less than 100 and
are coprime to 2, 3, 7 i.e X = 28
Alternate method to find X
The numbers coprime to 2, 3, 7 are coprime to 42.
The number of numbers less than 42 and coprime to 42
is

42 1

1
1
1
1 1
7
3
2

1 2 6
= 12
2 3 7

= 42

The number of numbers coprime to 42 in different


ranges are tabulated below

can be concluded that root (w) = Remainder

No.of coprimes

Range

Root (w) = 0 w is divisible by 3


The greatest value of w less than 500 divisible by 3 =
498 = 3(166). The least value of w less than 500 divisible
by 3 = 0 = 3(0)
Number of values of w satisfying the given conditions
= Number of whole numbers from 0 to 166 = 167.
Choice (C)

12
12

1 to 42
43 to 84
85 to 100
or 1 to 16

From 1 to 16, the numbers coprime, to 42 are 1, 5, 11, 13


X = 12 + 12 + 4 = 28

29

3n(26) will end in 6 when 3n ends in 1 and 3n(26)1 will


end in 5 and hence will be divisible by 5.
3n end in 1 which implies n must be of the form 4k
where k is an integer greater than 0.
n must be a multiple of 4 for the given sum to be
divisible by 70.
Choice (A)

The required number is 288 + 28 = 316


Alternate solution
The numbers coprime to 252 of 22(32)(7) are coprime to
2(3)(7) or 42. The number of numbers less than or
equal to 42 and coprime to 42 are

1
1
1
) (1 ) (1 )
2
3
7
1 2 6
= 42 =12
2 3 7

42(1

Solutions for questions 47 to 49:

Now 1108 = 42(26) + 16


In each group of 42 numbers 1 to 42, 42 to 84, 84 to
126, .. 42(25) to (42)(26) i.e. 1050 to 1092, there are
12 coprimes to 42.
Among the last 16 (ie., 1093 to 1108) or the first 16
numbers, i.e. 1 to 16 there are 4 coprimes to 42
(Q from 1 to 16, 1, 5, 7, 11 are the coprimes to 42 As 1
as well as 1093 are both! More than a multiple of 42
while 16 and 1108 are both 16 more than a multiple of
42, number of coprimes to 42 among the last 16
numbers = number of coprimes to 42 among the first 16
numbers.)
Choice (C)
43. Each coin will be flipped as many times as the number
of factors of the number denoting the position occupied
by that coin.
Since perfect squares have odd number of factors, the
coins occupying positions which are perfect squares will
be flipped an odd number of times and hence will show
tails at the end of the game.
The difference between consecutive perfect squares
increases as we increase the number. So, the greatest
difference in positions between two consecutive tails is
484 4411 or 42. The greatest number of consecutive
coins all showing heads at the end of the game is 42.
Choice (C)
Solutions for questions 44 to 46:
44. The coin occupying the position which has the
maximum number of factors will be flipped the
maximum number of times.
Number
Prime Factor
No. of Factors

210
2(C)(5)(7
16
240
24(C)(5)
20
5 2
288
2 3
18
15
324
2 2 24
Thus among the given coins, the coin occupying the
position 240 was flipped the greatest number of times (20
times).
Choice (D)

47. The unit's digit of the numbers and the unit's digit of
their cubes, are tabulated below.

Units digit
of the number
0
1
2
3
4
5
6
7
8
9

Thus if numbers are considered in sets of ten, the unit's


digit of the sum of the numbers and the unit's digit of
the sum of the cubes of the numbers are same.
Unit's digit of 13 + 23 + 33 + +103 = unit's digit of 1 + 2
+ +10 = 5. There are 9 of these 5s.
The units digit of the given sum =
The units digit of 5(9) = 45
So, the units digit is 5.
Choice (B)
48. Among the 99 numbers considered, the total number of
digits are as follows.
Number
total no.
of values
of digits
Single digit numbers
9
9
Two digit numbers
90
180
The digit 0 occurs 9 times and all the reaming digits
occur 20 times.
Sum all the digits = 9(0) + 20(1 + 2 + +9) = 900
The above sum when divided by 99 will leave a
reminder of 9.
Choice (B)
Probability
Solutions for questions 1 to 4:
1.

45. Coins which were flipped in round 13 were coins


occupying positions which were multiples of 13. Among
these coins, the coins which were flipped for the first
time after round 1 were 13 and the prime multiples of 13
(where the prime number is more than or equal to 13).
The coins flipped for the second time in the 13th round
were as follows.
13, 13(13), 13(17), 13(19), 13(23), 13(29), 13(31) and
13(37) i.e. 8 coins in all.
Choice (B)
n

n +3

46. Let S= (3 + 1) + (3 +2) ++ (3


1)
Number of numbers between 3n and 3n+3 = 3n+3 3n 1

S=

(3

n+3

3n 1
2

)[(3

) (

+ 1 + 3n + 3 1

{3 (26) 1}[3 (28) ]= 3 (14) [3 (26)1]

)]

For S to be divisible by 70, 3n(26)1 must be divisible


by 5.

Units digit
of its cube
0
1
8
7
4
5
6
3
2
9

If the smaller number is i, the bigger could be


i + 2, i + 3, i + 4 or i + 5.
Therefore, if the smaller number has any value from 1
to 45, there are 4 possible values for the bigger number
i.e., 45(4) or 180 favourable cases. The tokens could
also be (46, 48) (46, 49) (46, 50) (47, 49)(47, 50) or (48,
50). Thus, there are 186 favourable cases. The
required probability is

186 186(2) 186


=
=
C2 50( 49 ) 1225

50

Choice (B)
2.

S = {1, 2, . 105}
P = {1, 2, 4, 5, 7, 8, 103, 104}
Q = {1, 2, 3, 4, 6, 7, 8, 9, 101, 102, 103, 104}
R = (1, 2, 3, 4, 5, ,6, ,8 , 9,10, 11, 12, 13, .99, 100,
101, 102, 103, 104}
We have used the same symbols P, Q, R to denote
events as well as sets. But this doesn't cause any
confusion.
n(P) = 2(35) = 70
P(P) = 70/105 = 2/3
n(Q) = 4(21) = 84
P(Q)= 84/105 = 4/5
n(R) = 6(15) = 90
P(R)= 90/105 = 6/7

30

as 3, 5, 7, are pair wise co prime, we expect that the


events would be pair wise independent. We can verify
this. Suppose P has occurred i.e, x P = {1, 2, 4, 5,
103, 104}
we rearrange the elements as shown below
1
2

4
5

7
8

Coordinate Geometry
Solution for question 1:
1.

10 13 16 19 22 25 28
11 14 17 20 23 26 29 etc

Q 8 4
=
=
P 10 5

We see that p

whether P occurs or not, the probability of Q doesn't


change. We can similarly verify that P, R and Q, R are
also independent. But P, Q, R are not mutually
exclusive because X could be 3 or 5 or 7.
Choice (B)

9x2 y2 + 6y A=0, Consider the following equation.


{3x (y a)} {3x + (y a)} = 0
9x2 y2 + 2ay - a2 =0
Since 2ay = 6y, (or a = 3), amd A = a2 = 9,the given
equation would represent a pair of lines.
Hence the value of A is 9 and the two equations are
3x y = -3 and 3x + y = 3.
Solving these two equations, we get x = 0 and y = 3.
Therefore the point of intersection of the two lines is
(0, 3)
Choice (B)
Inequations

Solution for question 1:


3.

4.

On 25% of the days, Alsi arrived late and on 35% of the


days, he left early.
The minimum percentage of the number of days when
he did not work for the entire working day is 35% (When
all the late arrivals are included in the early departures)
whereas the maximum percentage is (when they are
disjoint) 25% + 35% = 60%
Therefore the value of P can be only value from (1
0.6) to 1 0.35. i.e. from 0.4 to 0.65.
Choice (D)
The data is tabulated below
Morning
Evening
P (Read)
0.10
0.30
P (Doesn't read)
0.90
0.70
The probabilities are tabulated below.
Morning
Evening
Probability
R
R
0.03
R
N
0.07
N
R
0.27
N
N
0.63
The probability that he reads the paper neither in the
morning nor in the evening is 63%.
Expected number of days =

63
(30) = 18.9
100
Choice (B)

Miscellaneous
Solution for question 1:
1.

There are 7 A sets. Each has 6 elements (not


necessarily distinct) It is more convenient to use the
word occurrences. There are thus 42 occurrences.
Each element of the union occurs in 3 sets.
Thus there are 42/3 or 14 distinct elements
Each of these 14 elements occurs in 6 of the B sets.
Thus there are 84 occurrences in the B sets. Each set
has 3 occurrences. Thus there are 28 sets.
Choice (A)

1.

x, y and z are the sides of a triangle


x + y + z > 0
Now x3 + y3 z3 3xyz = (x + y + z) (x2 + y2 z2xy yz
xz)
As x + y + z >0.
x2 + xy2 z2 xy yz xz = 0 [x3 + y3 z3 = 3xyz]
(x y)2 + (y z)2 + (zx)2 =0.
The sum of 3 perfect squares is zero, only when each
of them is zero.
x y = 0, y z = 0 and z x = 0
x = y = z.
Thus, it is an equilateral triangle.
Choice (B)
Statistics

Solution for question 1:


1.

When the numbers are arranged in ascending order, let


the fifth, sixth and seventh terms be t5, x and t7
When the greatest is removed, the median would be
t +x
(t5 + x)/2 5
= 10.5
2

On the number line, t5 and x are symmetrically located


to the left and right respectively of 10.5
Similarly x and t7 are symmetrically located to the left
and right respectively of 13.
The distance between t5 and t7 is 2(13 10.5) = 5
The median would be

t5 + t7
. While the difference
2

between t5 and t7 is 5, t5 itself can be selected at


random. The median of the 10 remaining numbers
would be t5 + 2.5 which therefore can't be determined.
Choice (D)

31

Vous aimerez peut-être aussi